Test of Reasoning

You might also like

Download as pdf
Download as pdf
You are on page 1of 402
arihant Verbal Reasoning Logical & Analytical Reasoning Non-Verbal Reasoning TION (e), @itsl@lae ie) s EASONING In All Competitive Exams Contains More Than 2000:‘Solved Questions rr ° Pre: : cre yoursmahbooh.wordpress.com C ontents Verbal Reasoning 1. Coding-Decoding a 2. Analogy &, 3. Classification & 4. Alphabet Test My 5. Word Formation Ry 6. Sitting Arrangement 58, 7. Direction Sense Test Te 8. Number, Ranking and Time Sequence Test &y 9. Puzzles 964 10. Number Series 107 11. Letter Series 2t-ty 12. Inserting the Missing Character 129419 13. Problems Based on Ages 138g 14. Blood Relations 144-459, 15. Clock and Calendar 160-1 ; is 16. Mathematical Operations and Symbol Notations 170-1 , 17. Logical Order of Words 5 18. Clerical Aptitude 1H 189-192 .. Logical and Analytical Reasoning . Logical Venn Diagrams 20. Eligibility Test 193-205 21. Syllogism 207-215 22. Statement ang Assumptions 216-205 226-283 Scanned by CamScanner yoursmahboob.wordpress.com 23. Statement and Conclusions 234-242 24, Statement and Arguments 243-251 25. Statement and Course of Action 252-257 26. Verification of Truth of the Statement 258-260 27. Data Sufficiency 261-268 28. Input-Output 269-282 Non-Verbal Reasoning 1. Mirror Image 285-291 2. Water Image 292-296 3. Series 297-317 4. Analogy 318-329 5. Classification 330-337 6. Paper Folding and Cutting 338-348 7. Grouping of Identical Figures 349-353 8. Formation of Figures 354-358 9. Counting Figures 359-365 10. Embedded Figures 366-371 11. Figure Completion 372-383 12. Figure Matrix 384-389 13. Cube and Dice 390-398, Scanned by CamScanner - yoursmahboob.wordpress.com Test of Reasoning & How to Crack It Skills in Reasoning are supposed to be the parameters of how analyticg and discerning the mind of an individuals I is seen that a person , high level of reasoning is quite successful and well adjusted to y surrounding environment, while a person with low reason, capabilities is not so successful in life and mal-adjusted to the environment around. This is why the tests of reasoning have bees devised to ascertain the suitability of a person for a job. Test of reasoning is a method of measuring the mental Capacities, i, includes tests to solve problems and arrive at rational answers and solutions ina logical way. Test of Reasoning in Various Competitive Exams There is no specific syllabus of Test of Reasoning. The range, dept, pattern and format of questions varies from one exam to another, Alsg the proportion of test items of reasoning out of the total items of the test changes from one exam to another. In Management Aptitude Test(MAn), there are 40 items of reasoning out of the total of 200 items while in different Bank PO Exams almost 50 items are there from reasoning outof the total 200. A comparison chartis given here: ‘ Total Numt No. of Questi S.No. Compettive Examination ofduestone Testa Teanga 1. Bank Probationary Officers 2.880 Geduae Level 3 MAT 200. ae entral Excise/IT Scanned by CamScanner yoursmahboob.wordpress.com Section 1: Verbal Reasoning 4 Verbal Reasoning Testis designed to test the ability to analyse the given information in words, and solve the problems using language-based reasoning framework. It aims at evaluating the ability to think correctly and constructively, rather than at simple fluency or Vocabulary power. It also is a test of intelligence, provides an assessment of individual's ability to think logically and solve problems in shortest of time. Verbal reasoning tests use words, letters and numbers and require logical reasoning and a reasonable knowledge of English language words. People generally assume that their verbal reasoning skills are relatively sharp because they engage in conversations and engage in at least some Treading of newspapers and magazines regularly. The differentiation between someone with poor verbal reasoning and someone with excellent verbal reasoning sill ies in one’s ability to read or listen critically, and to pick out some significant information, Si 'gnificant information can include known facts, specific opinions, statistics, sources and the like. This book How to Crack Test of Reasoning comprises ofthe following 18 chapters of Verbal reasoning 1. Coding-Decoding 10. Number Series 2. Analogy 11. Letter Series 3. Classification 12. Inserting Missing Character 4. Alphabet Test 13. Problems Based on Ages 5. Word Formation 14. Blood Relations 6. Sitting Arrangement 18. Clocks and Calendar 7. Direction Sense Test 16. Mathematical Operation 8 Number, Ranking and Time Sequence Test 17. Logical Order of Words 9. Puzzles 8. Clerical Aptitude Section 2 : Logical and Analytical Reasoning “Logical and Analytical thinking is not a magical process or a matter of genetic endowment, buta learned and cultivated mental prowess,” This book aims to make your logical and anal have these skills, and we use these mai lytical reasoning abilities more sharp. We all fact. ny times in our daily life without being aware of this All the competitive exam papers contain lo gical and mastering some tricks, and regular Practice of various sub-topical questions could solve analytical test items in the reasoning section, and these. Scanned by CamScanner voursmahboghaas HOARE C OH Bn, id almost allt! ss i Tr bon ngsategis and BIEN = eve 2. Eligibility Test nget in Diagrams 4, Statementand Assumption, “en 6, Statement and Arguments 3 crrementand Conclusions 6 a 5 Statement and Course of Action one ; 7. Statern 9. Data Sufficiency R . pal Reasonin, Section 3 : Non Vere danaive visual if nn . involves the ability 10 uné Te entifying relationships, similayi pebicns ing sl reasoning. For Se a dal sequences and relationghi-" recognisin aitlerences beeween ope enables candidates to analyze and solve complex objects, and remembering these. thout relying upon kills. As these tests don't require j, being limited by language s! se tests Ho ter = + i “ ry kK wo er abilities of those who have problems with reading and thinki cangive he re Teag ic it ific learning difficulties. In this seer k motivation, and those with speci n i mt sraiterntchoprers based ‘onnon- verbal problems asked in various competitive, exams, It comprise following chapters 1, Mirror Image 2 Waterimage 8. Formation of Figure 9. Counting Figure 2 Series 10. Embedded Figure 4. Analogy 11, Figure Completion 5. Classification 12, Figure Matrix 6. Paperfolding and Cutting 13, Cube and Dice 7. Groupingof identical Figures j [ 5 Tips fo Crack the Test of Reasoning 1 - Practice hard, x . Solve previous ; i andlevelot qaceeagtuestions of that exam to know the range, type 3. Exercise your memory so that all t | \ are always in your consciousness he important formula and methods 4. Practice for speed with Develop alertness and accuracy, Poise, \ cannes by CamScanner yoursmahboob.wordpress.com Verbal Reasoning yoursmahboob.wordpress.com Coding-Decoding Coding is a system of signals. This is a method of transmitting information in the form of codes or signals without it being known by a third person. ‘The person who transmits the code or signal, is called the sender and the person who receives it, is called the receiver. Transmitted codes or signals are decoded on the other side by the receiver—this is known as decoding. In questions on coding-decoding, a word (basic word) is coded in a particular way and the candidates are asked to code other word in the same way. The coding and decoding tests are set up to judge the candidate's ability to decipher the rule that has been followed to code a particular word/message and break the code to decipher the message. Order of the English Alphabet Theres] 2] a] rT oor PF rrr {1 | \ | Ttwtolealalstzluly|wlx| ez '26| 25] 24|23| 22|21] 20|19) 18) 17|16|15 rere reer | \ Here, are some methods/techniques are given bel low to remember the positions of English alphabets in forward or backward order. alalclolje|Fi[eiuli|s|x (a Scanned by CamScanner yoursmahboob.wordpress.com 4 How to Crack Test of Reasoning - Verbal 1. By using EJOTY and CFILORUX formulae, we can easily remembe, the letters of English alphabets. Pay, peppy $1015 3 2s CFILORUX EJOTY (sounds like 90'S namg 5 Jo oF lettey. FB x27, (sounds like a medicine name) ¥ * Backward order position of a letter = 27 - Forward order position ¢-9 Backward order position of B = 27 — Forward order position , Illustration 2. What is the Tumbe, SB * Bla, Mlustration 1. If CUP = 40, then KITE =? 7 (a) 10 (b) 20 (230 (d) 45 from right side? a 32116 (10 (b) 20 Solution (4) As, U P= 34.214 16.49 (025 (430 119205 « Simiary, KT PE 3114 94 204 5245 Solution (0) 27-G =27 ~7 =20 from, (using forward letter positions) meal em Sig! OFTES WEAN OY reer eg letters of alphabet in reverse order. a d XUROL YI Ff PeESPomxveot igre 5 10 15 2025 3 6 9 12 15 18 2194 Mlustration 3. If BAG =71, then VICE =? Similarty, Vil & @)69 = )70 (75 90 thug 5 18 24 29 ~5+ 18+ 244.2969 252620 Solution (2) As, BAG => 25+26+20=71 (using backward ite, Poti 3. If the sum of two letters is 27, then both letters are at opposite positi, ‘Some pairs of opposite letters can be remembered as given below | Post? Of Sach ot, DW MEW BY (By) 44+23= 27 2425 = 27 $424.5) G T (GTRoad) IR (indianRailway) 7 +20 =27 9418 =27 » iat Man Mlustration 4. In a certain code, LAKE is written as OZPV. How will BACK be in that Similarly, same code? (a) ZvxP (b) zvex aan (0) Yexp (a) YzPX l ‘ Solution (c) As, t 21s Litto $F ee LI 12+18=27 + Coes f L_{ 11+ 16=07 5422-97 Scanned by CamScanner yoursmahboob.wordpress.com Chapter 1*Coding-Decoding 5 Types of Questions Following are the categories/types of quest 5 it ie competitive examinations. jons which are generally asked in various Type 7 Letter Coding In this type, we deal with questions, in which th i aa hy q e letters of a word are replaced by certain orn letters according to a specific pattern/rule to form a code. You are required to detect the ig pattern/rule and answer the question(s) that follow, based on that coding pattern/rule. Illustration 5. In a certain code language, LTNIETAM ‘PICTURE’ is written as ‘QHDSVQF’. How would ‘BROWSER’ be written in that same code language? (@) Cawrps (b) CaPvTDs (©) CQPUTDS (¢) Cavepps (e) None of these INTIMATE Solution (b) Clearly, the letters in the word PICTURE are moved alternately, one step forward and one step R backward to obtain the letters of the code. Thus, we — have v el 1 ¥ p +4 Similarly, Pe c 1 E 1 tu rE a T +1 +1 A c—+D Of P R 8 1 =4s wis v 1 u thy sks 1 Milustration 7. In a certain code language, R -1 Q E |-1,, D ‘GIVE’ is written as ‘VIEG' and ‘OVER is written 1 as ‘EVRO’. How will ‘DISK’ be written in that e yr RES s same code? Milustration 6. In a certain code language, (a); SIOK {b) KisD (Q) KOS! (d) SIKD (e) None of these Solution (d) Here, ‘TTNIETAM is code for ‘INTIMATE’, then which of the following words has the code for “TREVNIETARBI? C. ved =O E (a) INVRETIBRATE (b) INVERTIBARTE 1 1 v 2 (c) INVERTIBRETA (d) INVERTIBRATE ‘ 5 P, 7 (e) INVERITBARTE - Similarly, _ . 3 Solution (d) The letters of the first half and the next half 1 1 of the code are separately reversed to obtain the es < a word. Thus, we have K O) jeu by Camocal 00: wordpress.com oy ing: ‘bal 6 How to Crock Test of Reasoning Vert tration Important Poi, de language, nts Dust 8. Ina certain code la gf setunone cai aey “BAT is written as +yzG’. How will sick’ not pecered a BEE written in that same code language” SSerequred 10 sohe it mathemancan , STE isuriten 2e3eine (omy (b) HRZP__(c) HR co Oe CASKET can be dena (d) RHPX (e) None of these eee end SPARK For a word in which a letter repeae Fotcerm repeats fOr 2nd letter in FasTE has code SZRSD, in this case," both cases So, the coding pactern be same forall che lecters Type 2 Direct Letter Coding In direct letter coding system, the code letter corresponding letters occur in the words. This is basi Miustration 10. In a certain codin ‘SHEEP’ is written as ‘GAXXR’ ang etters occur in the same se, ically a direct substitution met Illustration 9. In a certain code language, ‘STARK’ is written as ‘LBFMG’ and ‘MOBILE' is written as ‘TNRSPJ'. How will ‘BLAME’ be ‘HPXTN.. How can ‘SLATE’ be writ, written in that same code language? same coding system? a (a) TSFRI (b) RPFTJ (c) NUFTP (a) GPTNX (b) GPTXN (¢) GPXNT (a) TSFG) (e) None of these (d) PTGXN (e) None of these Solution (b) Using direct letter coding method, Solution (2) In both the words ‘SHEEP’ ang leterE is common and code for E's sin S—L and M—>T Hence, using direct letter coding method wd Tos O—N ' S—+G and 8B oF BR HOA c R—M Is Ex et KG L—p —E— x A =} = Similary, from terest codee i iy ua a Similarly, using the direct codes 8 —[R S— [6 L —lp A le aie woh A—|T — ly ~ |S —E—>|x Scanned by CamScanner yoursmanboond. WO Type 3 Mumber!Symbol Coding In this type of alphabetical code let, Mustration 11, ‘4-12-9-16, ‘WOMAN’? (2) 412414-26-13 (2 23-12:26-1413 (6) None of these Solution a) We have, If ‘WORK’ is coded as then how will you. code (b) 4-26-141312 (4) 23-15-13-1-14 W—4 "Hence, w—s [a O12 © he R—9 M— 14 kK—16 A— leo N— [3 Here, each letter is coded by the numerical obiained by subtracting its ‘position value’ in Englsh alphabetical order, from 27, €.g.,W, O. M, ‘A. Nare at 23rd, 15th, 13th, 1st and 14th postion in alphabetical order. So, their codes are (27-23), (27-18), (27-13), 27-1), (27-14), ie., 4, 12, 14,26, 13, respectively ie, their backward order position. Illustration 12. If ‘RAJU’ is coded as 11-12-13-14 and ‘JUNK’ is coded as 13-14-10-9, then how will you code ‘RANK’? (a) 9-10-11-12 (b) 10-11-12-9 (2 11-12-109 (@ 1211-109 (e) None of these Solution (c) We have, R11 A—12 JB U4 and J —> 13 U3 14 N— 10 kK— 9 Similarly, R —o[77 A—>/12 N—}10 K— 19 Here, each letter is assigned a specific value. Directions (Illustrations 13-15) Study the following letters and their corresponding digits codes following by certain conditions of coding and then Scanned by CamScanner Chapter 1-Coding-Decoding 7 questions, either numerical code es, signed word or ‘ers are assigned to the numbers. — ss ° ‘answer the questions given below them by finding. ‘out which of the digit combinations given in (a), (b), (c) and (d) is the coded form of the letter-groups given in each question and mark ‘your answer accordingly. Letters PINTAT | [RTE [elle ‘Digits/codes [5 {3 [9 [+ |41e[2 [71°18 Conditions, (OH both the first and the last letters in the group are vowels, both should be coded as $. (ii) If both the first and the last letters in the group are consonants, both should be coded as #. lustration 13. KUNAJB (@) 803917 (b) $0391 (©) #03918 (@) #03918 (@) None of these Mlustration 14. RBUKAE (a) #70892 (b) 670892 (©) 670982 {d) 607892 (€) None of these Mllustration 15. EBNAPI (@) 273954 (b) $73955 (9 #7395¢ (a) $73958 (@) None of these Solutions (\lustrations 13-15) We know that, in English alphabets A, E, |, O, U letters are vowels and remaining letters are consonants. 13. (@) From condition (i), KUNAJB beedde #03918 4 ()R BUKAE teidde 670892 Note This question does not follow any condition. 15. (b) From condition (), EBNAPI yhuuye $7395 yoursmahboob.wordpress.com B How to Crack Test of Reasoning Verbel Coding Type 4 Deciphering Message Word Coding/Mumer®! such In this type of questions to analyse A seed up The common code mbinations of two, the entire message can be t possi jrd/numeral can be found. ‘word/numeral are picked up. Proceeding similarly by picking uP # P ividua decoded and the codes for every individ Mlustration 16, Ina certain code nS, pit sit’ means ‘am boy; enit sit gil’, which of the following mee ‘gist? te (b) pit @ nit (e) None of these messages som an ore gn coos Hee See 17, in a certain code language, Miastration © “study very hard’, ‘958’ means 7a ek pays’ and “645' means ‘study and crate ich ofthe following i the code for “very? Type 5 Substitution Coding codes, any two messages bearing a common ie word/numeral will represent that word/code, (@) 8 (b) 6 (7 {d) Cannot be determined (e) None of the above Solution (c) Given, 7 BA——Ara\ vey [ha 0 °OD— Bl] om 5 As Oa, rt em ~ In the fist and second statements, the common Code citi ‘8° and the common werd is ‘hard ‘8! means ‘hard. In the first and third statements, ‘he common cade digit iss’ and the common wor is ‘study’. $o, °° means ‘study’ From Eqs. () and (i), 8 —s hard From Eqs. () and (i), 6 —+ study Hence, vey 7 In this type, some particular words are assigned with certain substituted names. Now, questions are formed based on that principles. Iustration 18. |f ‘white’ is called ‘blue’, ‘blue’ is called ‘red’, ‘red’ is called ‘yellow’, ‘yellow’ is called ‘green’, ‘green’ Is called ‘black, ‘black’ is called ‘violet’ and ‘violet’ is called ‘orange’, then what would be the colour of human blood? (a) Red (b) Green (0) Yellow (d) Violet (e) Orange Solution (c) We know that, the colour of the human blood is ‘red’ and given that ‘ted! is called ‘yellow. So, the Colour of human blood will be ‘yellow’ Scanned by CamScanner Mlustration 19. If Parrot’ is known as ‘Peacock’, ‘Peacock’ is known as ‘Swallow’, ‘Swallow’ is known as ‘Pigeon’ and ‘Pigeon’ is known as. ‘Sparrow’, then what would be the name of Indian National Bird? (@) Parrot (b) Swallow (@) Peacock (d) Pigeon (€) Sparrow Solution (6) We know that, Peacock is the Indian National Bird but here Peacock is known as ‘Swallow. So, the answer is Swallow. yoursmahboob.wordpress.com Let us Practice A. Base Level Exercise 1, In a certain code, SOBER is writti ten as RNADQ. How LOTUS can be written in that code? ISSC (Multitasking) 2013), (@) KNSTR (&) MPUWT (©) KMSTR (d) LMRST 2. If ‘MEAT’ is written as ‘TEAM’, then ‘BALE’ is written as {ssc (cGu 2013} (@) ELAR (©) EABL (©) EBLA @ EAB 3. If ‘WATER is written as ‘YCVGT’, then what is written as ‘HKTG’? (ssc (Gu 2013] (@) IRFE (©) FIRE (©) REFI (0) ERIF 4. If the word ‘TABLECLOTH is coded as ‘'XEMRANRIXT', how can ‘HOTEL’ be coded? TRB (ASM) 2011], (@) RIXaT (b) TIXAR (©) TAXIR (d) RAXIT 5. If ' CARING' is coded as 'EDVGKC' , and SHARES is coded as ‘UKEPBO', how will CASKET be coded as in the same code? [SSC (CPO) 2013} (a) EDXIBP. (b) EDWIAP () EDWPAL (6) EDWIBP. 6.If DEMOCRATIC is written as EDMORCATCI, how CONTINUOUS will be written in the same code? ICG PSC 2013] (a) OCTNNIOUSU (b) OTCNINUOUS (c) OCNTNIUOSU (@) OTNCINUOSU (e) CONNITUOSU 7. Ina certain code, P is #, Ais %, Cis ¢and E is @. How is PEACE written in that code? [BPS (Clerk) 2012] (a) #@%@# (b) #@#>@ (C) %#@H% @) #@%@ (©) None of these Scanned by Camscanner 8. In a certain code, 'BELIEF’ is written as ‘AFKKDH’. How would ‘SELDOM’ be written in that code? (a) ROKCHL (b) RFKENM (@) RFKFNO (@ TFKENP (@) None of these 9.In a certain code _ language ‘LIEUTENANT’ is written as 1232212021411420, then how ‘MANGO' can be written in that code language? (a) 13114715 (b) 1911474 (o) 14141975 (¢) 13114187 10. If ‘DELHI’ is coded as ‘73541’ and ‘CALCUTTA’ as '82589662', how will ‘CALICUT’ be coded? IMAT 20121 (@) 5279431 (©) 5978213 (©) 8251896 (¢) 8543691 11. If in a certain code, ‘DAUGHTER’ is written as ‘TERDAUGH', how will "APTITUDE! be written in that code? TSSC (Steno) 2012] (@) DEUAPTIT () UDEAPTIT (©) DUEAPTIT (@) DAUEPTIT 12, Ina certain code, ‘TERMINAL ' is written as 'NSFUMBOJ' and ‘TOWERS' is written as ‘'XPUTSF’, How is ‘MATE’ written in that same code? 118Ps (Clerk) 2012] (a) FUBN (©) UFNB (© BNFU (@ BNDS (@) None of these 13. 165135 is to ‘peace’ as 1215225 is to UB (ACIO) 2013] (©) love (@) aura (a) lead (©) loop If PARK is coded as 5394, SHIRT is coded as 17698 adn PANDIT is coded as 532068, how would you code NISHAR in that code language? (a) 266734 (c) 201739 14, (b) 231954 (6) 261739 yoursmahboob.wordpress.com 10 How to Crack Test of Reasoning - Verbal 15.If ‘SYNDICATE’ is ‘SYTENDCAI’, ‘then ‘PSYCHOTIC’ be written? (@) PsvICTCOH (©) PSYCOHTCI (©) PsicyocTH (@) PSICYCOTH 16. In a certain code, ‘REFRIGERATOR’ is Coded as ‘ROTAREGIRFER', Which words from the following would be coded as ‘NOITINUMMA’? written as how can (2) ANMOMIUTMI ——_(o) AMNTOMUIIN (©) AMMUNITION ——_(@) NMMUNITIOA (©) None of these 17, In a certain code, ‘CERTAIN’ is coded as ‘XVIGZRM! ‘SEQUENCE!’ is coded as ‘HVJFVMXvV', How would ‘REQUIRED be coded? [85 (CCL) 2012] (@) Fulwwir (0) VJIFWTRV (c) WJRIFV (@) IWFRIVW 18. In a certain Code, ‘BUILDER’ is written as JVCKSFE. How is ‘SEALING’ written in that same code? IRRB (TC/CC) 2009) (a) BFTKHOY (b) JOHKBT (C) TFBKHOJ () BFTKJOH 19. Ina coded language, BRINJAL is written as LAJNIRB. How will LADYFINGER be written in that code? SSC (Multitasking) 2014] (a) RNEGIFYDAL (0) RINEGIFYDAL (c) REGNIFYDAL (¢) RGENIFYDAL 20. In a certain code language, ‘CURATIVE! is written as ‘BSVDDUHS'. How ‘STEAMING is to be written in the same -ode language? (a) BFUTFMHL (b) TUFBFMHL (c) BFUTLHMF (d) BFUTHOJN (@) None of these 21. If ‘NEUROTIC’ = be written as ‘TICRONEU', how can ‘PSYCHOTIC’ be witten? (@) TICOCHPSy (b) TICCHOPsY (c) TICCOHPSy (d) TICHCoPsy veanicu vy Vaniucannicr a certain cog 22. Vr OMPUTRONE- is” wn ‘PMOCTUENOR’. Mon prin, ‘ADVANTAGES’ written code? tha (@) IDWJLAIC © Wouy, ot (©) WIDICLA @ uses (@) None of these 'GLOSSORY' is forded 23. If ‘GLOSS and ‘GEOGRAPHY’ = Soba, | ‘GEOLOGY’ can be codes (@ 915692 ) ors7s9p (©) 9057592 (© 815759, 24.If ‘REASON’ is coded as ‘BELIEVED’ as 7, what 2 no sy number for “GOVERNMENT ° ca @0o 6 4 a tie, 25. Ina certain code, ‘MOUSE: 'PRUQC’. How is ‘SHIFT © ng, same code? aya Bank (ct, (@) VIO () VKIDR gy ma (@) VIKRD_—_(€) None of these 26, Ina certain code ‘CALANDER’ as ‘CLANAEDR’, How is ‘crea written in that code? Lay (a) ICCRLURA (b) CRIUC, (©) CRIUCLAA (@) CRIARLCY 21. Ina certain code, ‘CLOCK’ ig Wt ‘XOLXP'. How will ‘Lotus’ a in that same code? (a) OGLFH () OLGFH () LOGFH (@) OLGHF (©) None of these 28. In a certain Code, ‘LATE! is iS writ ‘VGZO'. How will SHINE" be written; that same code? (@) VRMSH (b) VMSHR, (c) VMRSH (0) MVRSH (©) None of these 29. If LOFTY is co DWARF will be written eg EPFUY: then writen as (0) XSF oxeces eae 920 (©) DWESG (8) Nonw of meas”? PAT 30. In a coding system, stn ig ‘WQFQMN’. How can ‘GUEST’ be written in the same coding language? to) Farin ©) FPsv (9 FaSMN oF (6) None of these avoe 31. If in a code language, “PAR written as BDFGI ant che written as ‘MOXQUFGY', then how is ‘REPRINT’ written in that same code? (a) FQBFXIK (©) FGBUXJK (©) FGBFXGD (@) BGFKUK 32, Some letters are given below in the first line and numbers are given below them in the second line. Numbers are the codes for the alphabets and vice-versa. Choose the correct number-code for the given set of alphabets, CWEAZXIYKL 3957481026 JWXCLZ (b) 198264 (d) 197354 (a) 198364 (©) 198354 33. Some letters are given below in the first line and numbers are given below them in the second line. Numbers are the codes for the alphabets and vice-versa. Choose the correct letter-code for the given set of numbers. EMKBZWQ 591648 4297 (a) ZQMJDE (c) ZAMDEJ U 20 5 3 (b) ZOMEDJ (@) ZQEDM Scanned by CamScanner Chapter 1+Coding-Decoding 11 34. Some capital letters are given below in the first line and numbers are assigned to each of them in the second line. The numbers are the codes for the letters and vice-versa. MOEASITZ 35762940 ‘Choose the correct number code for the given set of letters EAST (a) 7620 (b) 7623 (e) 7624 (6) 7628 35. If DISC is coded as 8749 and ACHE is coded.as 3950, then HEAD is coded as (a) 5038 (b) 5308 (©) 3508 (d) 3805, 36. In a code language, ‘PRINCE! is written as ‘FLOWER’ and ‘PRINCESS' is written as 'FLOWERSS'. Which of the following word would be coded as ‘SLOWER’? (a) SRINCE (bo) SIRNCE (@) SANICE {@) None of these 37. In a code language, ‘ORGANISATION’ is written as ‘CBDWLQJWYQCL’ and ‘OPERATION’ is written as. ‘CXFBWYQCL'. How would ‘SEPARATION’ be coded? (@) EUXEBYQCL (b) JFQYWBCXOL (© JFXWBWYOCL = (d) QCLYWBFXJE Directions (Q. Nos. 38-41) In a defence message, GETAWAY, FIRE, BACK-WARDS, MOVE, SLOW is coded as BENCDCI, QHOE, PCTL-DCOXU, ZMWE, \VFMD. Based on this coding scheme, spot the codes of the following words 38. OVER (@) MWED. (b) MWEO (c) MWOE (d) MWZO. 39. DEADLY (a) XECXE! (b) XEEXCI () XECXFI (@) XENXFI ,voursmahboob.wagdpress.com Curt Test of Reasoning + 400 Kr WARD be Oreo ‘ 1 OTOCON atcsrar ” 1) BOENC BOLON 42019 4 cs stain code, "ZOOM" 1s written as ISMN and ROAD" 1s written ‘QOBE’ How would “NOMP* be coded in that conte anquaqee . b) QOHB : ) MONZ STC AML 1s written as ‘KWQKACJ’, 1 howe P| "pe written? How can °SPRINKLE be (b) ONPGLUC. (0) URTKPMNG 44.1 a certain code, ‘STOVE’ is written as NBLK’, then how will ‘VOTES! be wutten in the same code? [SSC (CPO) 2003) MEI MIG 1 HURBN (b) LBNKF UANBE (d) LNBKF 45.11 MACHINE" is coded as 19-7-9- 14-15-20-11, then how will you code DANGER’ in the same code? 8117-20 16-19-24 (b) 13-7-20-9-11-25 *O72013-19-24 A) 13-7-10-11-25 Directions 2 Nos. 46-50) Stuay the following carefully and answer the given NDB! Bank (PO) 2010), 22 certain code “318' means ‘tun very fast, 3 means to run away’, ‘97’ means ‘to see’ 3 means ‘very good’ 46. What 1s the code for ‘see’? 33 7 4 29 (€) None of these 47. Which of the foll good run away’? lowing represents, ‘see (a) 8472 (0) 7914 (©) 7319 8) 3289 (e) None of these ode for ‘run’? 3 (bo) 6 4 a9 (©) None of these Scanned by CamScanner the following 49. Wore to see away’? may i 7485 (0) 3149 0 09 © 2a79 {e) None of these 50. What does ‘4 FepTesent in ti aver (b) fast SoG, (D) goog ) un (6) none of these $1. If ‘LINGER’ is '123456" ang 5 and 'p, '56789', then ‘FIERCE’ wit) ye Rey, (@) 945667 (©) 4s6e79 (c) 345677 556789 © (e) Cannot be determined 52. In a certain code ‘PRISM: is ‘OSHTL' and ‘RUBLE? ig write ‘QVAMD’. | How "WHoRr? s written in that code i (@ XiPSM (b) VINSK Oy (c) UINSK (2) XGPQy 53. If A=1, ACE=9, then ART; Issc, @ 10 (0) 99 "ay (c) 29 (¢) 38 54, In a certain code, FLOWERs i, as EKNVDQR. How is spit written in that code? tra ya (@) TOOROLD (©) RTODALD (@ TODDROL (© RTOODIp 58. fA =1 ACT = 24, then FaT=7 185 0syy (2) 26 (b) 25 ©27 wy 56. If B=2,BAG=10, then BOX =2 [SSC (1043.9 @% 39 © de 57. If each of the letters in the Engi alphabet is assigned and even numet value by giving A=2, B = 4 and st what would be the total value off letters for the word ‘Lady’ wi similarly coded? [ssc (cova (a) 74 (b) 72 () 84 (0) 82 ene 58. If in a certain code language ‘ y 221848" then ‘how would SHADE" be written in that same code language? (a) a4t2s © 80215 (e) Base (@) eatz4 59. If ‘ACNE’ is coded as 1, 3, 14, ‘BOIL’ will be coded as *S then @)5,31.21,.25 &) 2,19.9,12 (©) 5.2019. 25 (8) 8, 29.19, 27 60. In a certain code language, ‘DOME’ is written as ‘8943' and ‘MEAL’ is written as ‘4321’. What group of letters can be formed for the code '38249'? [S81 Clerk) 2012) (b) MEDOA ( EDAMO (a) EOADM (c) EMDAO (@) None of these 61. If ‘FLARE’ is coded as 21, 15, 26, 9, 22, then how would ‘BREIF’ be coded in the same language? (@) 25,9, 22, 21,18 (©) 13,19, 11, 37,5 (©) 5.37, 11, 19, 13 (@) 25,9, 22, 18, 21 B. Expert Level Exercise Directions (Q. Nos. 1-5) Study the following information carefully and answer the questions given below it. ([IBPS (Clerk) 2012] Digits in the numbers are to be coded as follows pigits [9 [2[1]7]s[3le[s]s Codes | 8/V[M[L]o][PlAlF[R Conditions (i) If the first as well as the last digits are even, both are to the coded by the code for the first digit. (ii) If the first as well as the last digits are odd, both are to be coded by the code for the last digit. 1, 562183 (a) PAVMRP () PAVMRD (e) None of these (>) DAVMRD (d) DAVMRP Chapter 1+Coding-Decoding 13 62. If the word ‘LEADER’ is coded as 20-13-9-12-13-26, how would you write ‘LIGHT? (0) 20-16:17-15.27 (0) 20-15-16-10-28 (0) 20-17-15-16:28 (0) 20-16-15-17-22 63. In a certain code language, ‘SAFER’ is written as '5@3#2' and ‘RIDE" is written as '2©%#", how would 'FEDS' be written in that code? [RBI (Grade 89 2009), (2) 3#65 (©) 30%5 (©) 3#%5 (@ 3#%2 (©) None of these 64. If in a certain code language, ‘EAT’ is written as 318" and ‘CHAIR’ is written as 24156", then how ‘TEACHER be written in that code language? . Det Poice (Constable) 2005] (@) 8313426 () 8312496 (©) 8321436 (@) 8312346 65. If CAT = 12, then MAN =? @4 O24 ©@6 G15 (©) None of these 2. 627851 (@ PULRDM (b) AVLORM (©) AVLFOM (@) AVLROM (© None of these 3. 812354 (@) RLVPDF (0) FMVPDE () RMVPOR (@) None of these 4. 397416 (a) PBLFMP (b) ABLFMA (©) PVLEMA (6) PBLFMA, (@) None of these 5. 734192 (@) DPFMBV (b) LPAMBV (©) LPFMVB (6) LPFMBV (©) None of these canned by Camocanner yoursmahboob.wordpress.com How to Crack Test of Reasoning « Verbal 14 Ovrections 0) a eacn of the questions we raat toad By or Dan mee yncers numbered (2. JOT has have fd cut whch of the Lomaaers oven ty mpmrsents the group of the cash Aneaton the caving system and the conditions sever few Mork the ruber of that combanation 3s wir aemaee I nen of the combnatans correctly erent the grup of dts, mark (@), 1@,, None of Ren peo TEP “fel? [el LIE the first digit is odd and the last digit is ‘even, the codes for the first and last digits are 10 be reversed. 1) the first and the last digits are even, both are to be coded as %. tu) Ifthe first and the last digits are odd, both are to be coded as $. 6. 215349 (0) RAPK%& (0) CAPK%R 7. 671254 12) @MAO PX () sMacrs ic) @MAO Pe (d) %MA® PO €) None of these 8. 813469 1a) RAK®@D (0) DAK%@R ¢) DAPR@R (QD) RAK%@x ©) None of these », 794821 (a) MR&DCA (0) AR%DOM ‘c) M&ROGA (0) SRxDOS (@) None of these + $91426 {a) @RAKcP () PRAXG@ ©) @ARKEP (0) SRA%Ox (e) None of these Scanned by CamScanner ™ Directions (Q. Nos. 11-15) Study the foley information carefully to answer the gyf ‘questions BS cc? Ina certain code ‘ery large risk associate is writen a8 nu ta ‘tisk is very low’ is written as ‘gi se nu mi’, 4 “is hat also associated! is written as ‘a mi po fy: “inherent risk also damaging’ is writen a5 ty ny yu. E (All the codes are two letter codes only) 11, Which of the following represents ‘risk also large’? (a) nu tu po (0) nu giro () 0 pota (9) fu nu to (e)royutu 12, What is the code for 'very'? (a) ta (b) fu (ro (nu (@) gi 13. What is the code for ‘associated’? (@) mi (b) ta (10 Ogi (e) nu 14, What does the code ‘di’ stand for? (a) Either ‘damaging’ or ‘inherent’ (b) inherent (©) also (d) low (risk 15. Which of the following represents ‘that i low’? (@) po midi () se po mi (©) ta mipo (@) se po nu (@) ta mise 16.In a certain language, ‘sun shines brightly’ is written as ‘ba lo sul’, ‘houses are brightly lit’ as ‘kado ula ari ba’ and ‘light comes from sun’ as ‘dopi kup lo mo’. What are the codewords for ‘sun’ and ‘brightly’? (a) ba, sul (c) lo, ba (©) sul, lo (d) ba, lo yoursmahboob.wordpress.com 17. Ina certain code language, ‘po ki top ma’ means ‘Usha is playing cards’, ‘kop j ki ma’ means ‘Asha is playing tennis’, ‘ki top sop ho’ means ‘they are playing foot-ball’ and ‘po sur kop’ means' cards and tennis. Which word in that languages menas ‘Asha’? IGRPSC 2013) (@) ja (0) ma (©) kop (@) top 18. In a certain language, ‘me lo po’ means ‘anu weds vinay' and ‘pe to lo’ means ‘vinay comes here’, which word in that language means ‘come’? (@) pe (&) to (©) me (@) po (©) pe orto Directions (Q. Nos. 19-23) in each question below, a group of digits/symbols is given, followed by four ‘combinations of letters numbered (a), (b), (c) and (@). You have to find out which of the combinations (a), (b), (c) and (d) correctly represents the group of digits/symbols based on the following coding system and the conditions those follow and mark the number of that ‘combination as your answer. If none of the four combinations correctly represents the group of digits/symbols, mark (e) i.e., ‘None of these’ as the answer, [Vijaya Bank (Clerk) 2012] eee LP TTS]*]4]2]6)*] 7] 5] 8 bs tetters//6/E]P]A/K/OlF | Hla] 1]/R/a/u[m| v|T Codes Conditions (i If the first unit in the group is an even digit and the last unit is a symbol, both these are to be coded as the code for the symbol. (ii) Ifthe first unit in the group is an odd digit and the last unit is an even digit their codes are to be interchanged. ii) If both the first and the last units in the group are symbols, both these are to be coded as x, 19. @91$26 (@) JEFHRP (b) PEFHRP (©) XEFHRX (6) PEFHRJ (€) None of these Scanned by CamScanner 20. 21. 23. 24, 25. 26. 27. Chapter 1+Coding-Decoding 15 387#€9 (a) KMOTAE (b) KOMATE (©) EDMTAK (6) KOMTAE (e) None of these 4@3125 (@) VPKFRV (0) VPKFRI (©) XPKFRX (0) IPKFRV (e) None of these 1+ YoA1BT (@) QIFOMU (©) UNIFOMO (©) XIFDMX (d) UIFOMU (e) None of these 912486 (@) EFRIVS (0) JFRIVE (©) EFRIVE (@) XFRIVX (e) None of these In a certain code language, ‘123’ means ‘bright little boy’, ‘145’ means ‘tall big boy’ and ‘637’ means ‘beautiful little flower’. Which digit in that language means ‘bright’? @1 (o) 2 ©s 4 In a certain code, '975' means ‘throw away garbage’, ‘528’ means ‘give away smoking’ and '213' means ‘smoking is harmful’. Which digit in that code means ‘smoking’? @s os 2 @3 In a certain code, BASKET is written as ‘5$3%#1' and ‘TRIED’ is written as ‘14%#2'. How is ‘SKIRT’ written in that code? (BPs (PO) 2011) (@) 341 (0) 3e%41 (©) 3%#41 (@) 34% (©) None of these In a certain code language, ‘3a, 2b, 7c’ means ‘truth is eternal’, ‘7c, 9a, 8b, 3a’ means ‘enmity is not eternal’ and ‘4d, 2b, 8b’ means ‘truth does not’. Which of the following means ‘enmity’ in that language? (a) 3a (b) 7c (©) 8 (@) 9a (©) None of these yoursmahboob.wordpress.com 16 How to Crack Test of Reasoning -Verbel 28. If “Lily is called ‘Lotus’, ‘Lotus’ is called Rose, ‘Rose’ is called ‘Sunflower’ and ‘Sunflower’ is called ‘Marigold’, then which the national flower of India? (@) Ly (©) Lotus () Marigold (e) Suntower 29. In a certain code, the following numbers are coded by assigning signs 27273 ]eTs <|+ Lolt which number can be decoded from the given symbols? [SSC (Constable) 2012] (©) Rose als el? 3 (@) 63181 (©) 68731 (©) 62781 (6) 63118 30. On another planet, the local terminology for ‘earth’, ‘water’, ‘light’, ‘air’ and ‘sky’ are ‘sky’, ‘light’, ‘air’, ‘water’ and ‘earth’, Tespectively. If someone is thirsty there, what would he drink? (a) Light (0) Air (C) Sky (d) Water 31. If the animals which can walk are called ‘swimmers', animals who crawl are called ‘flying’, those living in water are called ‘snakes’ and those which fly in the sky are called ‘hunters’, then what will a lizard be called? (@) Swimmers (6) Flying (©) None of these 32. If ‘bucket’ is known as ‘tub’, ‘tub’ is known as ‘glass’, ‘glass’ is known as ‘saucer’, ‘saucer’ is known as ‘spoon’, then which utensil will be used for drinking water? (a) Tub (©) Saucer (c) Glass (©) Spoon _(e) None of these 33. If ‘orange’ is called ‘butter’, ‘butter’ is called ‘soap’, ‘soap’ is called ‘ink’, ‘ink’ is called ‘honey’ and ‘honey’ is’ called orange’, then which of the following will be used for washing clothes? (@) Honey (©) Butter) Orange (Q) Soap (€) Ink (b) Snakes (@) Hunters Scanned by CamScanner | 34, Ina certain code, the | 9 aed ina certain wat &y Sumbers ‘as follows by Rw | | ADILMNO | 1234567 5% Which word €@M Be decodeg 4. | lowing? nm ‘ 163514 97842 men (@) ANIMAL WORLD (0) ANIMAL LESS WORLD {o) WORLD OF ANIMALS. (@) ANIMALS WORLD ns (2. Nos. 35-39) Stuay y, Direc ation carefully and answer th, questions. 2 gy Ina certain code language. BY ‘economics isnot money is writen a ‘demand and supply economics is writen a ka’ "money makes only pat is writen a 2 gg ‘demand makes supply economics’ is wig mo ka ta’ a 35. What is the code for ‘money’ inthe code language? we os (@) ga (©) mo (©) pa @ta @la 36. What is the code for ‘supply’ in the, code language? (@) Only ta (©) Either pa or mo © Either mo or ta (b) Only mo (€) Only pa 37. What may be the possible code j ‘demand only more’ in the given cos language? (@) xine mo (0) mo zine (c) kine mo. (d) mo ziki (©) xikata 38, What may be the possible code for'wt and money’ in the given code languagé (@) pa gala (0) pa la lu (©) mo la pa (@) tulaga (©) pa lane yoursmahboob.wordpress.com Chapter 1+Coding-Decoding 17 39. What is the code for ‘makes’ in the given i) ‘bin cin vin rin’ means ‘drivers stopped all code language ? ins’ () mo (©) pa (ne (zi {@) ho . 40. ‘Drivers were late’ would be written as Direction (Q. No, 40) According to certain codes (a) min cin din (0 ‘min fin bin gin’ means ‘trains are always (©) cin cin fin late’. {©) fin din gin © gin hin min (ii) “gin din cin hin’ means ‘drivers were always (@) None of the above punished’. Answer with Explanations A Base Level Exercise LiAsS O 8 ER S.@AC A OR I bhp fehl | 2OKERT TEAM eS Then, BALE will be written as EALB 3 WATER sa 2] 2+] of B&iAD EMOCRATIC rit 1x 11x Now, word written for code HKTG is EDMORCATCI Foot RE ‘Similarly, Cc ONTINUQUS -a| -2| 2] =| ere X11 xX 1] 4. () As, OCNTN ItUOSU TABLE OTH 7. (d) Mf, ‘P! means #, ‘A’ means %, 'C’ means @ and TPPpEcr ese ". I: " ms XEMRANR I T E' means @. imilarly, HOTEL Then, P EA E Siew HOTEL Tite TIXAR #@%d@ Scanned by CamScanner SALTER Wy Wren yoursmahbo 18 How to Crack Test of Reaso! 8. (9 As, B > A Similarly. sor {| 17 | rin THT 5 H ap.amardpress.com eb F eur Lt—+k Lok nolo Are 1k pF 15.0485 7 e—+0 o—bN | | FH m0 Sy TeENDCA! , similarly, 9. (t) In the given code language, each consonant iS otss shownvas its place value. Further vowels have ps y CH an another sequence A>1E 92,1930 94U 95 MANGO - 13 11474 Bs foyeotH ie, 1311474 10. (9 Ay fT 16. (AS: Siriiaty @ PEE J on R A La} L4 E ° 7 and[C] [A] [¢] T F T A [4 Fe] Tf] a AU 2 3} [6) LJ L2) Then. i RON (OT G E 1 yy EGE BH E e T 3) (2 [15 )L1|La/Laiis R H I A R oO 11.5, ORGGITEA + fEADAGGA T FON 12345678 678 12345 ° E ‘Similarly, APTI TUDE. —»UDEAPTIT R R 12. (¢) If, Ea LNA Note All letters are coded in revese order. Al Ws rude ga wos CEET ESL ad = LOWER xvi@za ad $ QU EN + PEy gs AnRDSS HV JF VM Then, MAT Hens, F EQUI TE teyttrd %) TVJFRIV fasta Note The letters given here are opposite aw ? @ 2 6 e other. | | | | 18, (a) As, Similarly, 15 1 3 5 Bd 8.8 Place in th alphabetical order UD E F So, 1215225 is decoded as love. bexpalt oan = — 14.jP ARK SH D S TO UH EDSF NDS RANE eANy YExcz-3 Oe <«m yoursmahboob.wordpress.com 19. (@) Letters of the word are written in reverse order. 20. (0) As, Similarly a 48 S\H 48 uy Zs T F RPQY E uv a” No B T T D MF ' u i M Vv H N H E 's e7t NL 2.WASN EUR OT 16 LJ T crton ey LJ} L_] Similarly, P SyYc HoT ice La) T ("co H oP s y 22. (@) As, Simiarty, c po A ° Mo v M o ov D P cA A u TOON T toy Tos R — A s ° NG E N Oo G E RSs A 23. (b) o+—2 o_o e+—> oso a+» o+—2 os | I E 1 GEOLOGY = 9157592 24. (0) Given, REASON = 5 BELIEVED = 7 Here, Number of letters ~1 Now, GOVERNMENT = 9 Number of letter = 10-1= 9 p= Scanned by CamScanner Chapter 1-Coding-Decoding 19 25. (b) As, Ms P. Similarly, S*> V 0 =5R Hs kK uu 1 s 4a FD e—Sc TR 26. (0) As, ‘Similar, crc ce Asgt LPS ast Asan Coy n>Xa urxc OE LA e>cto reach RR RA 27. (b) As, 3121531 24 18 12 24 16 % t Similarly, 12 15 20 2119 bot us — l we+is-27 1541227 24 7=27 z+6=27 rer weom 5 SHINE Similarly, 14+ 13=27 ‘Os 127 a b00b.wordpress.com 20 How to crack Test of 31. 0) As [PB A—»0 (eose] [rox je] ey E—+G Swmilarly, = R—> F Du RF Ny EG PB RF 1x NJ ™T—+k Boiwxertz HL 33. (6) The correct lener 7 m code for the number is ZO MDE J given set of 40M OF sz 5 AG; Ee EAST = 7624 3 So, the correct number letters EAST is 7624” O° '0F the given set of Scanned by CamScanner a1 reasoning *Ver> 35. (0) mes J —~ code ‘TsTer 7 Tats TSN 7 HEAD is 5038. and Psp 37. (c) As[O—3C [R—=>B G—>D. NL Da S—>J A—W T—>yY I—>Q O-5¢ N—>L Similarly, yoursmahboob.wordpress.com Solutions (Q. Nos. 38-41) GB Ee TN A—c w— pd A—c Yo B—P A—c CT KL M—>Z and O—M vw EE 38. (6) 0 —f 3 D Zamoal[fxooomol[-nxom]lomss| Fa Iu Ro Ee w—p A—c R—o0 ox s—u Sv Lor OM W— Dd and R—>Q o—0 A—>B D—E Scanned by CamScanner Chapter 1+Coding-Decoding 21 Similarly, nm oo) M—IN Zz rp 44, (b) As, S—>F Similarly, V—> L TN o> 8 o—B TN vou EK EK SF 45. (c) As, M+, 19° Similarly, 0 +5, 10 at87 At$7 ct89 n+, 20 HS 14 6813 1845 e+844 n+8 20 R78 24 e284 Solutions (Q. Nos. 46-50) Means Code Ai —~ aA 200 — © fl oD — ‘A — AQ ow 46. (b) See —+7 47. (0) See Good Run Away —+ 8472 48. (b) Run —> 8 yoursmahboob.wordpress.com 22 How to Crack Test of Reasoning + Verbal 55. (c) As. A=1 49. s (4) Good to See away —+ 2479 She 3420024 52 )4— Good Smiary, FAT =6+1420=27 }. (e) As, L—+1 and F—35 B=2 ‘56. (0) AS. Ie o—6 aed BAGH2+147=10 N93 =F Similarly, BOX =2 + 15+ 24=41 G—s4 c—+8 57. (9 LADY =2(12 +14 4425) E45 e—9 22x42 = 84 _ R— 76 58. (0) As, O—>2 Similarly, S—)8 Simitary, F345 a! a4) I1—2 s—98 A—]1 E95 Ha p—l2 R67 E95 e—ls C98 59. (6) As, A—>1 Similarly, B —>| 2) E95 c—3 o—hs| Hence, cannot be determined. N14 1—]|9 52. (6) Given, £5 the POR ! Ss M 60. (d) As, ME and ME AL ler fer fr fer fs " pees Yrty 943 4324 oO Ss H T Lt emia, pe2 429 ad ou B LE tiid ble blob EDA o 61. (d) As, F—>21 Similarly, B—> 25 then ‘i A MD L— 15 R—9 .W ROL | | i [el A> 26 E22 er fer fer for for : A— 9 1 —+ 18 vin K E 22 F—21 53. (b) As,A=1 (place value) Note Ean, Foe and ACE =1+3+ 5 (place value of ACE) = 9 Fer kt TS coded in opposite alphaby Similarly, ART = 1+ 18+ 20(place value of ART) ” =39 8A Bs 1 4 5 18 54. () Given, hehe let FLOW ERS [= +8 [+8 [+9 [6 se fbb bbb fs 20 13 9 12 13 26 EK NV ODQR Similarly, 129 7 8 20 Thr, s UP R EM E . | on fT bbb bbb E pelos] [2 RTOQDLD 20 17 15 16 28 Scanned by CamScanner yoursmahboob.wordpress.com 63. () As, S45 and R42 A—@ I—0 F—343 D—-+% coo Ee R—s2 Similar, F—s3 E—+# D—% s—s 64. (0) As, E—+3 and C—2 at Ws Te A Is R46 B. Expert Level Exercise .@5 624 a @PPTle? PAVMRP [condition number (ii) follows) a 6278 1 mg tid z + AVLROM {no condition follows) 3098 12 54 OPltret RMVPODOR [condition number () follows) 4. 7 oreitit BLEMA [no condition follows} 5. 73 19 OT etl 3 LP MBV [no condition follows) 6. 274 4 @ tit 3 + i OA K %R {none of the conditions is applicable] Ve 71254 oF bidde wMAOP [condition (ii) is applicable] 8. (6) 1346 # f tidd ? DAK%*@R {none of the conditions is applicable} Scanned by Camscanner Chapter 1+Coding-Decoding 23. Similary, T—+8 E33 At c—2 H—94 E33 R—6 65. (a) As, 8 A P= (3+ 1+ 20)+2 =24+2=12 Simiary, AM = (13414 14)42 =28+2=14 roretttl SR%DOS [condition (ji) is applicable} wing Pet @RA% oun veo [condition (i) is applicable] Solutions (Q. Nos. 11-15) Or @EBD+OQGO @B@)»— @-OH Bl wai Seazeaids > Goo Ad wheat Bod saraaea—+> A@sis 11, (@) isk also large + nu fu ro 12. (@) very > gi 13. (6) associated + ta 14, (a) ‘di stands for either ‘damaging’ or ‘inherent’ 15. (6) that is low —> po mi se 16. (c) sunshines brightly —>>ba lo sul houses are brightly lt — kado ula ariba wo light comes from sun —+dopi kup fo mo (i) From Eqs. (i) and (i), brightly —> ba From Eqs. () and (i), sum —+ lo Hence, sun —+ lo and brightly —+ ba yoursmahboob.wordpress.com 24 How to Crack Test of Reasoning bal . (6) 3a2b Te —rtuthis eternal pas ae 7¢9a Bb 3a —+enmily is not eternal 3 144.2 8 —>truth does not footbat From Eqs. () and (i), 76 —tisleternal From Eqs. () and (i), 2 —» truth \ From Eas. (i) and (i), 8 —+ hot Hence, 8a —> enmity | Bee 18. @) molo po —sanu weds vnay 0 Jona ower of Ina is ig | pe tolo —svinay comes here w 28. (2 We know ne te caled Rose ia hay From Eqs. () and (i), lo —» vinay naniesew ite | Hence, come is either as pe oF to. 29. (b) According to given sigt % . | Woes 1 $2 =~ titi \ efitae rrigs | PEFHAS {po such condition is folom] 30, (a) Water quenches thirst and here water is cay, | Ue sie @Arooa 4 {(@ condition fotows) ag (Gemandjana orig} —> FRA] pa 24, (0) 123 —sbrgntitle boy 0 On 1.45 —stallbigboy (i) Leen aie only pant > BAR ne i) 37 —+ beau ite lower i orang From Eas. () ad (i), 1 07 era Ce fear ees From Eqs. () and (i), 3 —> tle + OHGE.m Hence, 2 —+ bright 35. (9 36.(e) —37.(@) 38.) 39.) 25.4 ssc oreeane. : (0) min fin bin gin —trains are awayslate ..() 2.13 —+smoking is harmful co) Sirished > SWers were elvan From Eqs. (i) and (i), smoking —» 2 bin cin vin rin —> divers stopped all trains 26.)AsB A S K ET (i : t : mg. din kin fin vin —+ all passengers were ¥ dT a | § 2 From Eqs. () and (jw), fin —+ ate 14°33 From Eqs. (i) and (i), cin —> drivers Smiay,S Ko | RT From Eqs. (i) and (v), din —+ were Piuvag Hence, drivers were late —» cin din fin 3% * 44 Y - Scanned by CamScanner rrr rm Analogy ‘Analogy means similarity or correspondence i.e, having similar features. In questions based on analogy, a particular relationship is given and another similar relationship has to be identified from the alternatives provided. Questions based on analogy are set up to test a candidate's overall knowledge, power of reasoning and ability to think. These types of questions cover every types of relationships that one can think. There are many ways of establishing a relationship like quantity and unit, worker and tools, cause and effect, word-synonym, word-antonym, country and capital, state and capital, country and currency, animal and the young ones (kid), male and female, animals and their resting places, games and places of playing, occupation their working place and their work. Here, some relationships are given, which are useful for solving questions based on analogy Country ital Currency Occupation. Working ___ Working Bangladesh Dhaka Taka Place Iraq Baghdad Dinar lawyer Court Tegal Practice China Beijing Yuan Servant House Service India Delhi Rupee Beautician Parlour Make up UK London Pound Mechanic Garage Repairing Japan Tokyo Yen Waiter Restaurant Serving USA Washington DC Dollar. Teacher School Teaching Russia Moscow Rouble Chef Kitchen Cooking Greece __Athens Euro Sailor Ship Sailing States of India Capital Male Female Sikkim Gangtok Dog Bitch West Bengal Kolkata Horse Mare Goa Panaji Bull Cow Tamil Nadu Chennai Cock Hen Odisha Bhubaneshwar Son Daughter Bihar Patna Brother Sister Scanned by CamScanner yoursmahboob.wordpress.com ‘j 26 How to Crack Test of Reasoning « Non-Verbal a ament Meas Individual Class Word Synonym = edividval_ Class Sphygmom- Blood pressure Frog “Amphibian “Assign Alot cee Rat Rodent Substitute Replace ph Earthquake Cup Crockery Abduct Kidnap seaman a Snake , Proud Barometer Pressure een = Haughty Prot ‘Ammeter Current Ostrich Bird oe dometer__Speed ' — LC eel Dearth ___Searcity_ Game Place of Stationery Playing ‘Animal Sound Word Antony Tennis Court Cock row Robust ‘Weal ‘Wrestling. Arena Duck Quack sale Boxing Ring Cat Mew ‘Chaos te Race ‘Track Cruel . Badminton Court Fi on aux Gentle Harsh ——_—__——~ ‘Owl Hoot Kind Cruel ‘Occupation Product Jackal How! tehagy Nee Farmer Crop Donkey Bray Mourn __Rejoice__ Architect, Designs ll = oak Producer Films —Quantity Unit Individual Dweling Teacher Education Power Watt Place Tailor Clothes Pressure Pascal Ton Den Curent Ampere spider web Ase Feectare: Horse Stable Mass Kilogram ied New Work Joule ing Palace Volume Lite Eskimo igloo Force Newton Types of Questions Different types of questions based on analogy that are asked in various competitive exams, have been given below ) Type 1 Direct/Simple Analogy In this type of analogy, we deal with questions which have three components. Two have some relationship and you have to choose from the alternatives which has the same relationship with the third component. Illustration 1. College is related to student in the same way as Hospital is related to ... (a) Doctor (b) Nurse (0) Medicine (d) Patient Solution (0) In the College, education is given to students, in the same way treatment given to Patient in Hospital Illustration 2. Major is related to Lieutenant in the same way as Squadron Leader is related to ... (a) Group Captain (b) Flying Attendant (@) Flying Officer (@) Pilot Officer Solution (c) Major and Squadron Leader are equivalent ranks in the Army and the Air Force, respectively. Same 35 Lieutenant and Flying Officer are equivalent ranks in the Army and the Air force. \ : =- canned by CamScanner yoursmahboob.wordpress.com Chapter 2+Analogy 27 Type 2 Completing the Analogous Pair In this type of questions, Way; Another word is also given. The candidate is required to find out the relationship between the first two words and choose the word from the given alternatives, which have the same relationship to the third word, as their is between the first two. Mlustration 3. Boat : Oar: Bicycle :7 two words are given. These words are related to each other in some lustration 4. Traveller : Journey : : Sailor :? (2) Pedal (©) Seat (a) Water (b) Ship (©) Road. (d) Wheel (© Voyage (d) Crew ‘Solution (a) Second denotes that part of the fist, on Solution (c) Second is the name given to the process of which the effon is applied to move it travel of the first. Type 3 Choosing the Analogous Pair In this type of questions, a pair of words is given, followed by four pairs of words as alternatives. You are required t. to choose the pair in which the words bear the same relationship as beared by words given in questions. Mlustration 5. Apostate : Religion (a) Teacher : Education (b) Traitor : Country (c) Potentate : Kingdom ‘Mlustration 6. Gland : Enzyme (a) Muscle : Spasm (b) Generator : Current (€) Organ : Kidney (d) Jailor : Law (d) Brain : Cortex Solution (b) Apostate is one who renounces Religion. —_ Solution (b) As, Gland produces the Enzyme. Similarly, Similarly, Traitor is one who betrays his Country. Generator produces the Current. Type 4 Choosing a Similar Word In this type of questions, a group of three words is given, followed by four other words as alternatives. The candidate is required to choose the alternative, which is similarto the given group of words. Mlustration 7. Kolkata, Mumbai, Mangalore Mlustration 8. Bhilai, Durgapur, Bokaro (2) Delhi (b) Lucknow (a) Baroda (b) Chennai (©) Cochin (d) Hyderabad (©) Chandigarh (d) Rourkela Solution (c) All are port cities of India, Solution (2) All cities are famous for steel plants. >cannea py Lamocanner yoursmahboob.wordpress.com = S 28 How to Crack Test of Reasoning - Verbal Some other Types of Analogy This types of analogies are explained as under (i) Number Analogy mbers by In this type of analogy, questions have- two numbers/group of nu ea ich have the same relatio, 9 @ ‘Rationship. There is also a third number/group of number whic "Ship from the alternative. You are required to find that alternative. Mlustration 9,2:8 (a) 20 (0 24 (b) 21 (@) 27 ‘Solution (4) Second number is the cube of first number in the same way 27 is the cube of 3. ie,29=8 =07 (ii) Alphabet Analogy In this type of analogy, first two groups of letters are related to each other in some w, are required to find out this relationship and then choose a letter group of letters related in the same way, as first two letters / group of letters are related, lustration 11. MUMBAI: LTLAZH : : DELHI: ? (@) COKGG (b) THLED (©) COKGH (@) BCKGH Solution (c) Each letter of the first group is moved one Step backward to obtain the corresponding letter of the second group, So, DELHI + CDKGH Note Thee can be anoter ype of analogy conta Beth, Number Analogy and Abhabet Ana ee Scanned by CamScanner Illustration 10.5:35::7 (a) 7:77 (b) 9:45 (11:45 (a) 3:28 Solution () Te fest number UIDIEd by thn, prime number to obtain the second number, and 7xM=77 7 and 11 are prime numbers. So, missing pair is 7 : 77. 8Y, You which ig Mlustration 12. NEWS : 14,5,23,19 : : PApy ER? (a) 16,5,16,1,18 (b) 18,5,16,1,16 (0) 16,1,16,5,18 (d) 32,2,32,10,36 Solution (c) Each letter assigned its position te So, PAPER > 16,1,165,18 ent Mlustration 13. FILM: 10 HOTEL : 12 15s (ig @) Solution (a) As, F 1 Loy “ ti gd 6+9412413 = 40 5 40+4=10 imilarly, Te . q Lo. 5 8415 +204 5412 = 60 60+5=12 yoursmahboob.wordpress.com Let us Practice A. Base Level Exercise 1. ‘Hare’ is related to ‘Burrow’, in the same way ‘Owl’ is related to ... (@) Hole (b) Hive (©) Nest (9) Barn Duma’ is related to ‘Russia’, in the same way as ‘Shora' is related to | (a) Malaysia (©) Afghanistan (©) France (d) Germany 3. ‘Konkani is related to ‘Goa’, in the sam way as ‘Dogri’ is related to |... (a) Madhya Pradesh (b) Odisha (©) Jammu and Kashmir (d) Gujrat 4. ‘Pitch’ is related to ‘Cricket’, in the same way as ‘Arena’ is related to ... (@) Tennis (0) Gymnastic (c) Badminton (d) Wrestling 5. Choose the option that expresses the same relationship as the word Tobacco : Cancer, has (@) Milk : Food (0) Bud : Flower (©) Soil : Erosion (d) Mosquito : Malaria (e) Honeybee : Wax 6. ‘Cyclone’ is related to ‘Anti-cyclone’, in the same way as ‘Flood’ is related to... . (@) Devastation (©) Havoc (©) River (d)_ Drought 7. ‘Doctor’ is related to ‘Patient’, in the same way ‘Lawyer’ is related to... [UP B.Ed. 2008) (b) Criminal (d) Client 8. As ‘Bald’ is related to ‘Blond’, in the same way, ‘Barren’ is related to (@) Customer (©) Magistrate [ssc (FCI) 2012} (b) Farm (0) Inhibited (@) Vegetation (0) Fertile ‘Scannea py CamScanner 9. ‘Atom’ is related to ‘Molecule’, in the ‘Cell’ is related to ... same wey) 28 treme aR (SM) 2006) (@) Matter (b) Nucleus (©) Organism (6) Battery 10. ‘Flower’ is related to ‘Petal’, in the same "Book' is related to ... meienieee [UC AAO) 2005], (©) Content: (6) Library (@) Pages (©) Author (@) None of these Directions (@. Nos. 11-13) in each of the following questions, there is certain relationship between two given words on one side of (::) and one word is, given on another side of (::) while another word is to be selected from the given alternatives having the same relationship with the word, as the words of the given pair bear. Choose the correct alternative. 11. Cougar : South America :: Okapi : ? (@) India (b) Central Africa (©) North America (6) Pakistan 12. Bow : Arrow :: Pistol: ? (@) Bullet () Gun (©) Shoot (d) Rifle 13. Cobbler : Leather (@) Furniture (©) Hammer (9) Chair Directions (Q. Nos. 14-22) In each of the following Questions, two words are given to the left side of the sign (::) and one word and a question mark are given to the right side of the of sign (::), select ‘one word from the given alternatives which bears the same relationship to the word given to the right side of sign (::) as that of the words given to the left side of the sign (::). 14. House : Door ::; Compound : ? (@) Gate (0) Fence (©) Foundation (6) Walt yoursmahboob.wordpress.com 3 0 How to Crack Test of Reasoning «Verbal 15. Hongkong : China :: Vatican : ? 25. Smoke : Pollution :: War: # [SSC (Mitashing) 2013) Peace Fr (a) (8 Canace en (> Meto (@) Rome (c) Treaty 16. Magazine : Editor :; Drama (@) Destruction ca [SSC (Mubtitasking) 2014] . 5; Tailor rpenter (@) Director (e) Payor i ane (©) Furniture 1" (©) Manager (0) Actor (©) Leather (6) Cloth - King : Throne :: Rider: ? (s8C #C0 20121 pirections (@. Nos. 27-33) In each of the foo (a) Chair (0) Horse pm ‘questions, there is same relationship between 16 (©) Seat (6) Saddle feo terms of the left of (::) and the sq relationship holds between the two terms 0 ing 18. Video : Cassette :: Computer : ? right. Aso, in each question, one term t0 the gy (a) Reels (b) Recordings Of (:.) fs missing. This term is given aS One Of thy (©) Files (8) Floppy alternatives, from the given alternatives defy (e) cru Gach question. Find out this term from the given, i 19. Plant : Seed :: ? : Bud ater (@) Leat (bo) Twig 27. Inthe English alphabet, ‘BDG' is to ‘Cry, (0) Flower (O) Fruit in the same way as ‘EGJ' is tot. (0) 20 20. Pleasure : Sorrow :: Right : ? @ FL (0) FM i [SSC (Constable) 2011] .) FIM (6) FIN (a) Wrong (b) Wondertul io tiene ofthooe (@) Happy (6) Sure 28. BFG : EJ :: RVW:? + Nis Warble :: Frog : ? . i 21, Nightingale : Warble = Frog :7 ayy a uw woe (a) Yelp (0) Croak (c) SWX (@) QUV (@) Cackle (8) Squeek 29. AFHO : GBDJ :: CHFM : ? (@) None of these [SSC (FCD 201) 22. Tanning : Leather :: Pyrotechnics : ? a Sa a a (@) Bombs (0) Fireworks (c) Wool (a) Machinery 30. HCM : FAK :: SGD :? Directions (Q. Nos. 23-26) In the folowing questions, (@) Q&B (b) a1B find the word which holds the same relation with (© ESQ (0) GES the third word as there in between the first {WO 34, GREAT : 25 :: NUMBER : ? [ssc (ch 201 (@) 36 (o) 38 23, MODERATE: INTENSIFY : NOMINAL : ? 2 (@) 24 (a) MEMORIAL (bo) EXPENSIVE (© DISTANCE (@ CHAOS 32. AHOP : CKSU :: BIMR : ? [SSC (CGU 2013 (@) EZUQ (&) DMaw 24, Oxygen : Burn :: Carbon dioxide : ? (©) DOKM (@ CaWN 10BPS (Clerk) 2012] (2) Isolate (©) Foam 33. AZCX : BYDW :: HQJO :? (c) Extinguishes (0) Explode (a) GREP (b) IPKM (©) IPKN (@) GRUP (€) None of these Scanned by CamScanner yoursmahboob.wordpress.com Directions (Q. Nos, 34-41 Chapter 2+Analogy 31 ) 'In each of the following questions, there is a certain relationship between two given ‘numbers on one side of (.: paces found from the :) and one number i: given on the other side of (: :) while another number is to be given alternatives having the same relationship with this number as the numbers of the given pair. Choose the best alternatives, 34.1,2,4.7:3,4,6,9::7:2,3,5,8 ISSC (10+ 2) 2012) (@) 0.13.6 ©) 2.4.5.8 ©1347 0) 3.5.6.8 35. 63: 21::27:7 (a) 6 9 1 3 36. 12:30: 18:7 (a) 36 (&) 42 (©) 44 (©) 45 37. 25 : 6253: 35:7 Issc CGu) 2012) (a) 1575 (b) 1205 (©) 875 (©) 635 B. Expert Level Exercise Directions (Q. Nos. 1-7) The following ‘each other, followed by four the original pair of words. 1. Surgeon : Scalpel (@) Musician : Instrument (©) Carpenter : Cabinet (©) Sculptor : Chisel (¢) Baker : Oven 2. Horse : Hoof :: ? (a) Man : Foot (b) Dog : Black (©) Paise : Rupee (@) Pen : Pencil (Mar 2012) [SSC (DEO & LDG 2012) 3. Which of the following has the same relationship as Sailor : Compass :: 7 IPNB (Clerk) 2011) {a) Student : Exam {b) Doctor : Stethoscope (©) Pen : Officer (A) Painter : Artist (e) Bricks : Plumber 4. Cells : Cytology :: ? (@) Worms : Ornithology (b) Insects : Entomology (c) Disecses : Physiology (@) Tissues : Morphology IRRB (ASM) 2009] Scanned by Camscanner : 1: 9638 :? 38. 6524 : 6465 :: 96: —_— aa (a) 9625 () 9736 (©) 9697 (@ 9579 (©) None of these 39. 100: 121 :: 144: 7 {SSC Multitasking 2014) (@) 160 (©) 93 (©) 169 (@) 426 40. 08 : 66 :: 7: 38 [SSC (10+ 2) 2013] (2 6 (© 12 @ 19 41. 583 : 488 :: 293: 7 (@) 581 (&) 201 (©) 387 (¢) 487 questions consist of two words each that have certain relationship between lettered pairs of words. Select the related pair that has the same relationship as 5. Chair : Wood :: 7 [SSC (10+2) 2013), (2) Book : Print (0) Mirror : Glass (©) Plate : Food (@) Purse : Money 6. Nuts : Bolts [cLat 2013} (@) Nitty : Gritty (©) Bare : Feet (6) Naked : Clothes (@) Hard : Soft 7. Book : Author [evar 2013} (a) Rain : Flood (©) Light : Switch {c) Symphony : Composer (@) Song : Music Directions (Q. Nos. 8-11) In each of the following questions choose that set of numbers from the four alternatives sets, that is similar to the given set. 8. Given set : (7, 77, 140) (@) (8, 80, 166 (©) (@, 88, 160) 9. Given set : (32, 24, 8) (@) (26, 32, 42) (©) (24, 16, 0) (0) (9, 88, 180) () (10, 110, 300) [SSC (CPO) 2005} (b) (34, 24, 14) (A) (42, 34, 16) yoursmahboob.wordpress.com x 3 2 How to Crack Test of Reasoning « Verbal 10. Given set : (8, 3, 2) (a) (10. 6. 5) (b) (63. 8, 3) {c) (98, 24, 5) (d) (168, 15. 4) 11. Given set : (56, 52, 36) (a) (95, 90, 65) (o) (85, 80. 50) (©) (60, 70. 99) (0) (45, 55. 65) Directions (@ Nos. 12-15) Jn each of the folowing Questions, choose that word which has the same relationship among the given three words. 12. Bhopal, Panaji, Gandhinagar (@) Amntsar {b) Alanabad (©) Patna (0) Baroda 13. Yuan, Kyat, Lira (a) Dwmam (b) Turkey (o) Maar (6) Mass 14. Yellow, Blue, Red (a) Black (b) Orange ( Wnte (6) Paint 15. Peat, Bituminous, Lignite (a) Coke (b) Granite (c) Basalt . (a) Anthracite 16, In this pyramid if 1122 31 12212, 1221:7 1 23 4 98 7 6 § to 1 12 13 «14 15 16 25 24 23 22 21 20 19 18 17 26 27 28 2 30 31 32 33 34 3 (2714 (0) 8 13.20 (o) 615 18 (a) 1023.30 17. Peacock : India :: Bear : ? (a) Australia (b) America (c) Russia (6) England Answer with Explanations A Base Level Exercise 1, (‘Burrow is the dwelling place of ‘Hare’. Similarly, ‘Barn’ 1s the dwelling place of ‘Ow'’ 2. (6) Russian’ parliament is called ‘Duma’. Similarly, ‘Atghanistan's’ parliament is called ‘Shora’. (Konkan is the language of ‘Goa’. Similarly, DDogh isthe language of Jamimu and Kashmir 4. (d) Crcket’ is played on ‘Pitch’. Simiaty \Wresting’ is done in ‘Arena’ 5. (0) As, Tobacco is a cause of Cancer. Smal, Mosquito is the cause of Malaria, 6. (2) The words im each pair represent opposite ‘conations 7. (d) Doctor’ deals with ‘Patient. Similarly, ‘Lawyer’ ceals win ‘Cient 8. (0) AS. Bald 's related to ‘Blond’ in the same way, Barren’ is related to Fertile’ 9. (c) Fast constitutes the second, As combining ‘Atom’ we get ‘Molecule’ in the same way combing ‘Cell we get ‘Organism’ Scanned by CamScanner 10. (a) ‘Flower’ is made of ‘Petals’. Similarly, ‘Book ig made of ‘Pages’ 11. (0) As, Cougar is found in South America. Similarly, Okapi is found in Central Africa 12. (@) As, Arrow is released from Bow. Similarly, Bullet is released from Pistol. 13. (0) As, Cobbler uses Leather to make shoes Similarly, Carpenter uses Wood to make furniture, 14. (a) As, Doors are for Entry in house. Similarly, Gates are for entry in compound. 15, (d) As, Hongkong is in China, Similarly, Vatican isin Rome 16. (a) As, Editor is related to Magazine. Similar, Director is related to Drama. 17. (0) As, King sits on the ‘Throne’. In the same way ‘Rider’ sits on the Seat, 18. (d) Second is recording device and first is visual device. yoursmahboob.wordpress.com 19. (c) As, grown form of seed is Pi rly, gt £5, grave orm ol seed is tent. Silay. Grown 20. (o) Pleasure’ is opposite of 'Sorrow’ in the same way ‘Right’ ts opposite of ‘Wrong’ 21. (0) As, sound of Nightingale is Warble. Similarly, the Sound of roy e Croan Swath 22, (6) As, for finishing the ‘Leather’, Tanning. Method {8 Used in the same way. ‘Pyrotechnics’ is used for tre works.” 23. (6) MODERATE and INTENSIFY are antonyms and NOMINAL and EXPENSIVE are antonyms. 24. (c) The fire Burns due to presence of Oxygen and the fire Extinguishes due to Carbon dioxide. 25. (d) Pollution is the result of Smoke. Similarly, Destruction is the result of War. 26. (b) As, Tailor makes Dress. Similarly, Carpenter makes Fumiture, 27.0 a2 BDG—+CFI Similarly, code for EGJ is given as 2 — Gur IM we 2 67 990 BEG:E!S Se a aed 18 2 23 RVWw i = 29. (d) As. A fe c Similarly, @ 8 Oo J Lele b | 1 0 BH 30.(a)H C Scanned by CamScanner Chapter 2+ Analogy 33 31. (@) In word GREAT, the number of alphabets = © GREAT = (5) =25 In the same way, in word NUMBER, the numiber of alphabets o wuss: =(6F = 36 32. (b), 4 bloke bees 3, AZCX Haso BOR aw TERA BAAS, 1 2 35, (b) 63+3 =21 2743=9 36. (b) 12:30:: 18:42 co” ost xo46 x2e8 37. (d) As, 25 is common in both 26 and 625 and 6 is added Similarly, 35 is common in both 35 and 635 and 6 is added 38, (d) 6524-59 = 6465 9638-59 39. (c) 100 = 10: 121 144 =12%: 13° (b) (08) + 2 = 66 (06? + 2 = 38 41. (9 As, 5+8+3=16) | 4+8+8=20) Similarly, 2+9+ ral +4 34+8+7=18 yoursmahboob.wordpress.com 34 How to Crack Test of Reasoning - Verbal B. Expert Level Exercise 1. (¢) Scalpel is used by Surgeon during operation, in the same way, Sculptor uses his Chisel for crating sculptures, 2. (a) The Hoots of an animal such as a Horse are the hard lower parts of its feet. n the same way Foots are the lower parts of Man. 3. (0) ‘Sailor is judge the direction through ‘Compass* ‘Same as ‘Doctor is judge the breathing and heart beats through 4. (8) ‘Cytology’ the science of study of ‘Celts’ nthe same way ‘Entomology’ is the Science of study of insects’ 5. (0) As, Chair is made of Wood. Similarly, Miror is ‘made of Glass. 6. (¢) As, Nuts are covered with Bolts. Similarly, Naked is Covered with Clothes 7. (0. Book is written by Author. Similarly, Symptiony is composed by Composer. 8. (0 As, 7 7 7 140 : Tet = 7x20 Similarly, 6 08 8 LJ: axti 8x20 9. (Here, 32 24 B LiL “8-16 Similarly, 24 16g. L_4tL_y -8 -16 160 Scanned by CamScanner 24 10. (6) Here. e 3 2 Similarly, et (Gen, 56 52 36 1. (@) 4-4? Then, 95 90 65 L_4lL—_+ -5 -8? 12. (All are the Capitals of indian States. 13. (a) Al are the currencies of different Counties, 14, (0) All are the colours of rainbow. 15.) 16. (6) 1 34 2 Q@OQ7 65 10 GAB 14 15 16 25 24 23 hed 19 18 17 2% 27 28 29 30°G) @® 93 34 a5 % {n this pyramid, you can see the patte So, the correct option is 8. 13,20 17. (@ As, Peacock is the National Bi A rd Similarly, Bear is the National, Animal of Russet yoursmahboob.wordpress.com | Classification Classification means ‘to assort the items’ of a given group on the basis of a certain common quality they possess and then spot the stranger o- ‘odd one out’. These questions are based on words, letters and numerals. In these types of problems, we consider the defining quality of particular things. In these questions, four or five elements are given, out of which one does not belong to the group. You are required to find the ‘odd one’. Note There may be possibility of more than one logic to be applicable but we need to apply optimum and best logic of all possibilities. There are several types of questions tliat can be asked on classification in different catagories. Some of the types are discussed below Type 1 Choosing the Odd Word In these type of problems, some words are given which belong to real world. They have some common features except the odd one. You are required to find the ‘odd one out’. Directions (\ilustrations 1-3) /n each of the following questions, five words have been given ou: cof which four are alike in some manner while the fifth one is different. Choose the odd one. 1. (@) Pear (b) Apple (©) Litchi (d) Guava (e) Orange Solution (e) Out of given fruits orange is citrus fruit. So, it is different trom others. 2. (a) Deck (b) Quay (Q Stern (d) Bow (e@) Mast Solution (b) All except quay are parts of a ship. 3. (@) Tomato (b) Gourd (©) Brinjat (d) Cucumber (e) Potato Solution (e) All the vegetables except potato grow above the ground level. Vuaiiied Uy Vana: yoursmahboob.wordpress.com 36 How to Crack Test of Reasoning Verbal Type 2 Choosing the Odd Pair of Words ss of SOME comp, | ba: In this type of classification, different pairs are classified on *e features/ properties like names, places, uses, situations, origiD. © words are BIVEN Out of of won which the wor Directions (llustrations 4-6) In each of the following questions, fiveltour Pe La Ve words in fourlthree pairs bear a certain common relationship. © differently related. of heavy. But in othe, itis antony™ a 4. (a) Gold :Omaments _(b) Cloth : Garments Solution Snore . (©) Wood: Furniture (d) Leather : Footwear (b) Ink: Pen (e) Earthen pots : Clay (a) Petrol:C2 G) Lead : Pencil Solution (e) Except pair (e). in all other pairs, the first 'S (@) Garbage : Dust {o) inal other Pars ty the raw material used to make the second. solution @, emt wn (or its functioning. * requ 5. (a) Broad : Wide (b) Light : Heavy (©) Tiny: Small (d) Big: Large Type 3 Choosing the Odd Letter Group of letters are given. one out of them 1S different any a In this type of problems, some grou! this is need to identify as the relevent Answer the group of feters which i lifferent from others. Directions (ilustrations 7-9) Choos ea od (b) EKS (@ Re 7.(0) H wa + te) PB (e) RvB (oT @z =1,B . Z = 26, then Solution (t) All other leters except (0), occupy the Solution (c) Taking ‘even-numbered positions in the English alphabets. G-+Q=7-G4+2=1 K-€ +5 =11-6+5=1 8. (a) GE (b) MK R- (J +6) = 18- (10+ 6) =2 fo wu (@ Qn Y-(P +8) =25-(16 + 8) =1 somoncrrenc ew Kyo oH eee a FoL Vv PO Type 4 Choosing the Odd Number/Pair of Numbers In this type of classification, certain numbers/pair of numbers are given, out of which except ) cone, all have some common characteristics and hence are alike, The ‘different one’ is to be chosen as the answer. Directions (illustrations 10-14) in each of the following questions, fivelfour numbers are given. Out of these ‘ourlthree are alike in a certain way but the fifth one is different. Choose the one which is different from the rest fourithree. 10. (a) 2 (b) 32 AL. (@) 57 (b) 87 (9 131 (© 56 (@ 128 (@) 133 (e) 143 Solution (c) Each of the numbers except 58, can be Solution (c) Except 131, all other numbers are non-prime expressed in terms of powers of 2 (composite) numbers. Scanned by CamScanner yoursmahboob.wordpress.com Chapter 3+ Classification 37 (b) 28:32 (@) 14:16 (@) 54:62 12. (a) 144 (b) 169 (a) 288 (e) 324 Solution (2) Except 288, al oer numbers are square of natural numbers, As, 12? = 144 (©) 256 14, (a) 21:24 (d) 70:80 1 Solution () Here, 31 13, (a) 125 (b) 216 and at (a) 525 (e) 343 Except option (e) every ratio is equal i, 7/8. Solution (2) Except 625, all other numibers are cubes of natural numbers, (9 729 Let us Practice y A. Base Level Exercise Directions (Q. Nos. 1-30) in the following questions, three/four out of the fourrfive alternatives are same in a certain way and so form a group. Find the odd one that does not belong to the group. 1. (@) Unicom (©) Rhino ——_(¢) Fox 10. (2) Sky (©) Star (©) Planet (@) Antelope —(e) Jackal (@) Comet —_(e) Moon 2. (a) Swimming (0) Sailing 11. (a) Rigveda (b) Yajurveda () Diving (@) Driving (c) Atharveda (@) Ayurveda (e) Boating [EPFO 2011] (e) Samaveda_ 3. (a) Blackmail (©) Smuggling 12, (a) Frequency polygon (c) Snobbery (@) Forgery (0) Rectangle (@) Sabotage (©) Bar 4. (@) Gaton (Ton (A) Pi {88C (Steno) 2013] (c) Quintal (@) Kilogram 13. (@) Silicon (©) Platinum [SSC (CPO) 2012] (©) Arsenic (@) Antimony 14. (a) Carpenter (b) Goldsmith, 5. @ ee (a Pane (6) Blacksmith (d) Driver [88 Coy 2013) [SSC (Constable) 2012] 15. (a) Misdeed (b) Corruption 6. (a) Herd (b) Flight (c) Failure (@) Offence (©) Hound (2) Swarm [SSC (Steno) 2013] ©) Wiley 16. (a) Rain (©) Shower 7. (@) Ring (6) Tyre © Plate (©) Sieet (@) Raisin (0) Bangle (e) Rubber tube [SBI (Clerk) 2011] 1Z. (a) Rickshaw (b) Taxi (©) Tanga : (@) Cant ©) Phaeton (a) H Ti He sat ag. @ many 18. (a) Brass (b) Gun metal (©) Bronze (4) Germanium 9. (a) Cheese (b) Wine [SSC (Steno) 2013) (©) Mik (@) Curd Scanned by Camscanner yoursmahboob.wordpress.com 38 How to Crack Test of Reasoning - Verbal 19. (a) Mountain (b) Valley (6) Glacier (@) Sea-coast (@) Ridge 20. fe) Graph (0) Chat) Model (6) Drawing (e) Figure ——_—*{1BPS (Clerk) 2012] 21, (@) Mountain (b) Hill (©) Plateau (@) Plane [SSC (1042) 2013] 22. (a) Asia (b) Australia (©) America (@) Africa —_(@) England 23. (a) Celio (b) Guitar (0) Fute (0 Violin [SSC (Maltitasking) 2012] 24, (2) Sweetness (0) Elegant Brignt (d) Beautiful ° (© SC Oultasking) 2072) 25. (a) Cholera (0) AIDS (©) Cancer (a) Heath (@) Jaundice (LIC (AAO) 2011] 26. (a) Chop (©) Sit {c) Chirp (@) Sice 27. (a) Political Science (b) History {(c) Philosophy (@) Physics [SSC (CPO) 2013] 28. (a) Author (b) Novelist (6) Poet (@) Publisher [SSC (1042) 2013) 29, (a) Barter (b) Purchase (@) Sale (0) Borrow [Ss¢ (1042) 2013), 30, (a) Scury (b) Rickets (c) Night-bindness (0) Influenza SSC (CGU 2013] Directions (Q. Nos. 31-50) In each of the following ‘questions, some groups of letters are given, all of which, except one, share a common feature while ‘one is different. Choose the odd one out. 31. (a) A (DE 1 @z @u ° 32. (a) A () D F K @T ® 33. (a) MORV (o) CEHL. (c) CENT (@) JLos [SSC (Cat) 2013) Scanned by CamScanner 34, (a) doba (©) spa 35. (a) GMS (a) LAK 36. (a) POXZ 8 MNPR: 37. (a) DACB (©) SPOR 38. (a) MOR (d) ACF 39. (@) HKL (@) MON 40. (a) BdEQ (d) Twxz 41. (a) CdaB (0) RsqP 42. (a) FUL (¢) OGHJ 43. (a) ABDG (@) GHUK 44, (a) ABYZ () EFUV 45, (a) DFIMR (¢) GILPU 46. (a) HIN (@ asw 47. (a) BOFH (o) SUWY 48. (a) COFE (@ wx 49. (a) NKHE (©) KHEB 50. (a) 3216 (©) 3208 ™ (b) zw (@ hofe Issc Milian @ sg sy a {Copeaton Da oy, (b) EBCD (@) XU © oh ©) sux 6 Uw (© cro ) Pu (8 Kmiy () WU ©) Ly (@) HitG * (b) RUVX (@) NPOS woe (©) EFHK ) BPS (Ce ay (b) COWX (@ GHTV [RRB (ASM ann, (b) CEHLO (QHUMPT [SNAP any oe (©) PRU {PNB (Clery 20 (b) Moas (o) ze (o) KLM () STU (@) HK TLIC (ADO) 2013 (o) MDA ( wraN [SSC (10+2) 200) (b) 2398 (@) 2015 {ps 2011 om yoursmanvood.wordp. Chapter 3-Classification 39. Directions (Q. Nos. 51-64) In each of " bat oon ne following questions, fvelfour numbers are given. Out of these fourlthree ae athe in crate way but one cere: Chose the one ich aeret Wehtoe te tore 51. (@) 9.49 ©) 13,121 o ", oer © 7.25" psctcroyr0 SET 4S BI (7B 52. (a) 14, 17,23 (0) 19, 22, 28 SB. (a) 527 (o) 639 (c) 246 (a) 356 (©) 17, 20, 26 (¢) 21, 23, 30 59. (a) 17 (b) 44 (©) 20 13 ' oa 2013) 60. (a) 729 (b) 123 (©) 423 (6) 621 (a) 4967 5 58 8 6243 8 on 61. (a) 24 (b) 60 (©) 124 () 210 kB Group Dy 2017) G2.(a) 25 W)9 1G 1B 54. (a) 272 (&) 210 (c) 240 (0) 304 {SSC (Multitasking) 2013) Bsciccv 20131 3, (a) 120 (©) 168 (© 290 55. (@) 46, 57 (©) 38, 49 (@) 380 (@) 728 (c) 41, 52 (@) 64,73 ips aes eres (a) 1 56. (a) 144) 169 (6) 1968 (a) 210 (6) 195 (©) 267 B. Expert Level Exercise Directions (Q. Nos. 1-15) in each of the folowing questions, fourive pairs of words are given, out ofthese words ‘one pair does not bear the common relationshio which rest bear. You are required to find that odd pai. 1. (@) Needie-Prick (6) Gun-Fire (©) Auger-Bore (4) Chise-Carve ISSC (0 & DEO) 2012] 2. (a) Lion-Roar (0) Snake-Hiss (C) Bees-Hum (@) Frog -Bleat (©) Dog-Bark (NBR (ler 20111 3. (@) Dim : Bright (0) Wrong : Right (c) Shallow : Deep (0) Genuine : Real (© Involvement : indifference 4. (@) Oil: Lamp (0) Water : Tap (©) Oxygen : Life (@) Power : Machine 5. (a) Circle : Arc (0) Line : Dot (0) Hexagon : Angle (@) Square : Line [SSC (10+2) 2013] 6. (a) Captain-Team (b) Boss-Gang (c) Prime Minister-Cabinet (0) Artist-Troupe [RRB (Group ‘D’) 2012) 7. (a) Hard-Sott (b) Pointed-Blunt (C) Sweet-Sour (d) Long-High (€) Day-Night [LIC (ADO) 2012} ‘Scanned by CamScanner 8. (a) Ice cube : Cold (0) Iron : Hard (¢) Marble : Smooth (@) Purse : Money ‘9. (a) Petro-Car (0) Electricity-Television (©) Ink-Pen (d) Dust-Vacuum Cleaner (e) Pen-Paper 10. (2) Day : Night (©) Across : Along 11. (@) Flag : Flagship (©) Court : Courtship (c) War : Worship (@) Friend : Friendship 12, (a) Tree : Stem (b) Face : Eye (0) Chair : Sofa (0) Plant : Flower 13. (a) Light-Heavy (©) Shott-Long [UC (AAO) 2012} (©) Up: Down (@ Small: Large (©) Crime-Blame (@ Man-Woman [eps 2011) 14, (a) Principal : School (0) Soldier : Barrack (©) Arist : Troupe (@) Singer : Chorus 49 Yoursmahboob.wordpress.com FW to inst echoed) 2012 weer st 4 THO Ips (Mutttaking 2 Basis o for aged GY ST100 “_ S wr me 2001 [snap 20111 - Answer with A Base Level Exercise 1. (@) Except. ‘Unicom’, all animals. But Unicom is 2. (@ Except ‘Driving’, all activities are related to water but Driving is related to road. others are real ‘an imaginary animal. 3. (c) Except ‘Snobbery’, all other words are crimes, ‘Gatton’ is different because this is use fOr 4. (¢) Only ‘are used for solid quid measurement and rest measurement. 5. (d) Platinum is literent from all others because it is very costly and generally not used to make utensils except all other are used to make utensils. 6. (c) Except ‘hound! all represent group. Scanned by CamScanner Oo ~ Test of Reasoning « Vertut aah ” ay je 16 qoare a : fy 2124 az. ets (a 6 ma 2 oats * ie) , win 28. wo N68 wm anos are p) 9:12.16 ‘ bat 29, OS ‘ BO TS 1 we IS8€ tena 30. Fart out the set among) the four Se whichis ty FO. Te gNen 80 . ine onenn G3: 39097) 1S8CU00 2 yp (es 13°20) (oy (7: 15:22), er 2-3 fo) 12 19:25) ‘among the tour sets Which ig yg BAL Fined out the the gan set . . IS8C 1052) 9p) (13:20:27) ws 18; 25: 92) ay (B11 18) (oy ( ee OL tne four sets Which is tig 32. Find out the set among the given set Given set is 19) (a) a8 1) fe) (12: 24: 36) ISSC (1062) 299) (b) (10: 20 : 26) (d) (80 : 36 : 42) 12 Explanations 7. (©) Except ‘Plate’, all things are hollow body. 8, (0) Except ‘Heavy’. all others are used to denote hy size. But ‘Heavy’, denotes weight. 9. (0) Wine is alcohol while others got by animal 10. (0) Except ‘Sky’, all others belong to the same class ie., celestial body. 11. (@) Ayurveda is the branch of medicine. All others are Vedas. 12, (b) Frequency polygon, Bar and Pi and differet types of graphs while rectangle is a geometica figure. 13. (b) Silicon, Arsenic and Antimony are conductors while platinum is an element y sem yoursmahboob.wordpress.com 14, (@) Except, ‘Driver’, all three can made their specialisation: but only ‘Driver’ serve his Worle 15. (c) Except ‘Failure’, all others are ilegal activities, 16. (d) Here, all options are related to rain or except Raisin. The meaning ot “Raisn partially dried grapes which is different from all others, 17. (0) Taxi’ is auto-driven whereas other it either man or animal driven. ems are 18. (4) Brass , bronze and gun metal germanisn an elemen, > SYS wie 19. (¢ Except ‘Glacier’, all other are mad While Glacier is made of ice. mee ies 20. (0) Except ‘Model’, all are ort Me same because all are 21. (@) Except ‘Plane’, all tems have height. 22. (@) Except ‘England’, all others are continents 23. (@) All are music instruments. Except flute, all have string to play the music but flute does not have 24. (0) Allexcept sweetness are related with beauty but sweetness is related with taste 125. (d) Except ‘Health’, all other are different kinds of diseases. 26. (¢) Except ‘Chipp’, all other are related to cutting So, the odd one is chirp which means sound of birds. 27. (d) Political Science, History and Philosophy are the subjects related with humanity while physics is a subject of science. 28. (d) Except the ‘Publisher’, all other dre related to “iterature’ 29. (d) Except ‘Borrow’, all the options are the terms of business. 30. (4) Except ‘influenza’, all other disease are caused by lack of any vitamins 31. (d) Except 'Z’, all others are vowels. 32. (a) Except ‘A’, all others are consonants. 33. (c) Except option (c), all options follow the pattern a3 MO R VC E H t Lat Ltt owe ve Eee Cc & NTS LO 8 L4Ltlt L4+Ltlt 2 wT From above, it is clear that word CENT is different from ail others, Scanned by CamScanner Chapter 3+Ciassification 41 “Li LL Land Lila 35. (c) In all other groups, first and third letters are Sas See le ener estoy ares inermioh rane 36. (b) Except option ‘b’, all are same because all other Poses a ets ng othe gap. 37. (a) Option (a) shows different nature from the rest bores o—24a—24c— 98 e248 —1.c—150 $_ op 1sQ—214R x2 putt 38. (¢) In all other groups, there is a gap of two letters between second and third letters as they appear in English alphabet. 39. (a) In all other groups, there is a gap of one letter between second and third letters as they appear in English alphabet 40. (¢) in all other groups, there is a gap of one letter between first and second letters as they appear in English alphabet 41. (@) In all other groups, third and fourth letters are consecutive alphabets. 4a2d@F ft J LR U Vv xX z Lill LAs 2 ST LiL LL oe eT 43.0 42 42 ty Cy A poGco FI L+ceL+ut+ ut 7 oe on os a2 2 ry Ty € FH K GH J kK Lttt Lt Lt 7 oS mo ‘Theretore, GHJK is different from the other thre a2 ‘ I) HOW to Crack Test of Reasoning + Verb! are same because mn Mose ae "anc forth and seco" 4s. 46. ret PRL a omer nawng same gap but 2+3+8-3>10 3205-2 3+ 245-010 2015-3 240+ 5-156 Therefore, 2015 1s different from the other three. 49. 0) Sli) 9-227 + Pea 139-2=11 4 1 2121 10-228 + 8 7-2=55 § S20 144 3217 417+ 6=23 19+ 3=22 4224 8 Expert Level Exercise 1. (@) Only this 1s not an instrument action pair. 2. (dj Frogs don't leat, they croak 3. (2) In all otner pairs, the two words are antonyms to each other - (6) In all other pairs, second requires the first to Continue the corresponding function 5. (¢) Hexagon is not made from an angle, 6. (0) Arist is just a part of a troupe. 4. Scanned by CamScanner oursmahboob.wordpress.com 17+ 3= 20-4204 6-25 Bit 2= 2342347290, 444 B+ O47 = 25, 53. ( $067 Which i omy, om, 5555 +54 9+ 9+ 9= 20 whigy 5 6243 +6 +24 443-15, whey 5. 6655-76 + 6+ 545222, vp divisible by 5 rk 22+74+2=11 54.(0) Ff +14+0=3 2402+ 4+0=6 304=3+ 0+ 4=7 sum of digit of all number prime number where as su) CF Gigit is anon-prime. Monee, Aso others. 55, (d) Except 64-73 all are having ai Except 210, all others are squ, 56. (0) ers. ‘are 57. (0) Except 28, all others are multiples oy» iI other numbers, the sum of 58. (0 ts ig equal to the third cig ty, 59. (b) Except 44, all are prime numbers 60. (2) Only 729 is a perfect cube. 61. (Except 124, allother numbers are muti, ature any My My 62. (d) All except 18 are square of nz (6? =25 (3) =9 (4° =16 63. (d) Each of the numbers except less or one more than the si number. 64. (e) Except 267, all other numbers are one leg, the square of a certain number. . 380, is quae dees 7. (d) Except 'Long-High’, all options having os of second word. 8. (a) In all other Pairs, second denoles Characteristics of the first. 9. (d) Except ‘Dust-Vaccum Cleaner’, all sex things are come in use with help of fist. 10. (¢) Day-Night, Up-Down and SmalkLarge ¥ Pairs denote the opposite relationship an” them. But across and along are synonym. WC MUnLOOD Wort ieee con 11. (©) War and Worship are not interrelated ie wresavant 12, (¢) In all other pairs, second is a part of the first. 13, (0) Except ‘Crme-Blame’, al three options having opposite words. 14, (@) In all other pairs, second is a collective the frst mee 15. (0) In all other pairs, second is a product obtaned from the first. 16. (c) Except "4-32", in all options second number having cube of frst number. 17, (a) Except '80-9'. in all options first number is square of second number. 18. () Except (6). in all other groups the second umber is the square of the first number (d Except (¢), in all other groups the second umber 1s the cube of the first number. (a) Except option ‘(a)’, in all options first number imide by second number. B+3=11 149-4 749-10 343-645 22. (a) Except 8-15, in all the options, both of th e numbers are the square of any natural number. 23. (¢) Except option (d) difference ot all other numbers are divisible by 2. 24, (d) Only ‘option (d) second number is twice of first number. 19, 20. 21.14 Scanned by CamScanner Chapter 3+Classification 43 25.9 5a 9 n—*4e7, 2468 ‘So, '48-68' is different trom other three. 26, (0) 5-223, 19-16 = 327 -23=4,31-28 9 27. (0) nal others, middle number is the sum of other {wo numbers 28, (b) Except "14,11,12', all remaining options having ‘only even numbers. 9 12 16 29. (0) o 3 8 . Lit util se ae 17 20 24 S51 54 5B Lt Lut at Bos Hence, 0-3-6 is different from other three. (0) A, wi 7 30. (@) As, 23:30:37 = WF Stary 6 3m er0- LG 31. (b) As, 19 + 7 = 20-+20 + 7 =27 Similarly, 18 + 7 =25 +25 + 7 = 32 32. (0) As, 62 1B L4L_+ Te 8 ‘Similarly, army 12 24 36 riers yoursmahboob.wordpress.com Alphabet Test | isa test of the sequence of words in alphabetical order as thy abate other specific way, appear in the dictionary arranged in som is test requires the knowledge of English alphabets and way to use dictionary. ‘Questions based on alphabet test is based on orderly arrangemen, of different words or to detect a rule in which they are arranged. Som. questions can be based on finding a letter or number following a particular ry}, in a alpha-numeric sequence. Important Facts about Letters 1. Position number of letters in English alphabet ABCDEFGHIJKLM bet be debe tbudve 123 4 5 67 8 9 1011 12 13 NOPQRSTUVWXYZ phedbbebu ddd 14 15 16 17 18 19 20 21 22 23 24 25 26 2. Position number of letters in English alphabet in reverse order ABCDEFGHIJKLM bes be beburuauay 26 25 24 23 22 21 20 19 18 17 16 15 14 NOPQRSTUVWxyz bebe ese burragy 1312111098765 4324 veanicu vy Vanucannicr yoursmanboo Chapter 4+ Alphabet Test 45 3. We know A. E, 1, 0 and U are consonants of English aiohaiee Vo*e's of English alphabet and remaining letters are tan Mia to Mi bettors are called the first half of English alphabet > © Z) letters are call ® second half of Engli 6. To the len ae ites ar ale the second half of English alphabet 7. To the right means A + Z (A to Z) 8. From A > c i From A a2 a to Z) letters are called left to right of English alphabet prez fo A) letters are called right to left of English alphabet Per the position of the opposite letter of English alphabet A-Z AtoZorAaza BY By-By = Scan-X-RAY D—W Dew vening F—U First U (You) GT Road H—S High School Indian-Railway J-Q = Jack-Queen Kevin Peterson L—O Light ON Man Types of Questions There are four types of questions based on the alphabet test which are generally asked in various competitive examinations Type 1 Arrangement According to Dictionary Arranging words in alphabetical order implies ‘to arrange them in the order as they appear in a dictionary’. For this arrangement, first we shall take the first letter of each word and then arrange the words in the order in which they appear in the English alphabet, then take the second letter and so on. Illustration 1. If the following five words are arranged in alphabetical order, then which word will come in the middle? Illustration 2. Arrange the given’ words in alphabetical order and choose the one that comes at the second place. (a) Electric (b) Elector (2) Bathing (b) Banking (c) Electrode (d) Electrode (©) Backing (@) Banishing (e) Electron (€) Barricading Solution (a) Arranging the words in alphabetical order Solution (d) Arranging the words in alphabetical order, we have Elect, Elector, Electric, Electrode, Electron. we have Backing, Banishing, Banking, Barricading, Hence. Electric will come in the middle. Bathing Hence, Banishing will come at the second place. Scanned by CamScanner yoursmahboob.wordpress.com 46 How to Crack Test of Reasoning - Verbal Type 2 Alpha-Numeric Sequence In this type of questions, a jumbled given, Sometimes only English alphabet Directions (ilustraions 3-7) Study the following arrangement carefully and answer the questions given below. F@39H@ADI%4ERSMK 2URPSW681TJV7 Llustration 3. How many symbols are there in the above arrangement, which is preceded by a number and followed by a letter? (a) None (b) One (Q Two (d) Three (6) More than three Solution (2) No symbol is preceded by a number and followed by a letter. Illustration 4, Four of the following five options are alike in a certain way, based on their positions, in the above arrangement and so form a group. Which is the one that does not belong to that group? (a) 0%O (b) 58R (© Tvs (d) AIH (e) E%S. Solution (e) Except 'E % $', all follow same rule/patterm, Mlustration 5. If all the symbols from the above arrangement are dropped, which of the following will be the twelfth from the left end? @4 kK @2 wu (e) None of these Type 3 Letter-Word Problems In these type of problems, a word is like — how many letters in the word is as far Scanned by CamScanner sequence of some letters, numbers and symbol, from A to Z are given, ae Yy Solution (c) Atter droping all the symbols, the r arrangement will be ett, FOSHADISEMKE] ‘So, twelfth from the left is 2. Illustration 6. How many consonant; there in the given arrangement, whic, preceded by a number and not foiowed letter? ss (2) None (b) One (0 Two (a) Three (©) More than three Solution (©) According to the quEStON, in the ge arrangement, two such consonants are there yp H © and 5 W 8 whch are preceded by aman and not followed by a letter. Mlustration 7. Which of the following is y tenth to the left of the sixth from the righ, end of the given arrangement? (a) % kK (€) None of these Solution (b) Required element = (6 + 10) = 16th from the right =s RPSWBITyy, (b) $ (a) P given and candidates are required to answer questa h a rat qu the begining ofthe English alphabet. Sometimes. a work ooo ng Of the word as, its fon many letters remain same in its position, if they a a word is given and they are asked, which vowel a word is given and then they are asked how re arranged in alphabetical order. Sometimes, 1 or a consonant going to follow it, yoursmahboob.wordpress.com inatration 8. How many such pairs of letters ere an the word ‘CHANNEL, which has ny letters between them in the word as in the English alphabet? (@) None ‘ti One () Two (@) Three (e) More than three Solution() CH AN NW g So, such number of Pairs are AC and LN. Miustration 8. How Many such letters are there word PMCLNOB' which remain samein its position, if they alphabtical order?” * *@nged in an (a) None (b) One Mc) Twe (@) Th (e) More than three ee Type 4 ‘Rule Detection In ‘these type of problems or are asked to choose the correct Iilustiation 11. Find out the acljacent letters in the series, is two? (2) LORTW (b) Gimps (©) KNPSV Solution (6) G Jo oM pos +o FS FSF Chapter 4+ Alphabet Test 47 PMc LIN 0] 8 B ct M\Nlo} P So, such type of letters are N and O. lustration 10. if each vowel in the word ‘GLADIOLUS’ is substituted with the next letter of the English alphabetical series and each consonant is substituted with the letter Preceding it. How many vowels are present in the new arrangement? Solution (c) (@) None {b) One (2 Two (d) Three (€) None of these Solution’a) GLADIOLUS bhiddeuag FKBCJUPKVR So, no such vowel is present in the new arrangement questions, a certain rule is given in questions and candidates alternative that follows this given ru'e. Correct alternative in which numbers of letters skipped in between (d) EHKLO Let us Practice 4. Base Level Exercise Directions (Q. Nos. 1-3) in each of the following questions, five words are given. Which of them will come in the middle of all, if arranged alphabetically as in a dictionary? 1. (a) Cruise (b) Crupper —_(c) Crusade () Crude) Crumb 2. (a) Minisulate (b) Minimalis (c) Minority (6) Miniature (e) Ministerial 3. (a) Sentinel (b) Sentimentally (©) Sententious (@) Sentence (e) Sentimentalize >canneg vy Vamocanner 4. Arrange the given words in the sequence in which they occur in the Dictionary and locate the last word. {ssc (CPO) 20111 (@) Frankenstein (b) Frankincense (c) Frankalmoign (d) Frauendienst 5. Which one of the following words will appear fourth in the English Dictionary? [SSC (Steno) 2012) (0) Encourage (a) Encounter (@) Encradle (©) Encroach yoursmahboob.wordpress.com z 48 How to Crack Test of Reasoning» Verbal ji ch pairs of letters are t 6. Which will appear fourth in the English 14, How many such Paie Doc) Men, m Dictionary? [SSC (FC 2072] fu ae jetters between them a, ch (2) Quick (b) Question the English alphabet? " (©) Quality (©) Quit sine ) One (©) Two Directions (Q. Nos. 7-11) In each of the following (9) Three (@) More than three | Questions, arrange the given words in the i | sequence in which they otcur in te detorary 18. How many, ae een eet | and then choose the correct sequence. in ne wore teters between them got 7.) Bound (i) Bonus the English alphabet? (iii) Bunch (iv) Board fa) None (b) One 10) Two (a) (.G4.4i Gi) ©) Gi)... (@) Three () More than thre © M..0.H 46. How many such pairs of letters are 8. (i) Aqueous (ii) Aquarium in the word ‘JOURNEY’ each of which i tic has as many letters betweer. ‘hem in th. | (i) Acquiline Gs) Acme rom 2003) word (in both forward and backwary | @ w.00,00.0 Ai) v) directions) as they have between them in © @.69.00.6) Dwi) the English alphabetical order? 9. (i) Preposition (i) Preparatively (a)None —(b) One Oto" (iii) Preposterous (iv) Preponderate (6) Tee (@) More than three Isse cou) 20111 7, How many such pairs of letters aire th (2) (.00.0.6i) (©) 0.0.6.4 es " . ee oun © 69.00.0409 in the word ‘STREAMING’ ach of . which has as many letters between the 10, (i) Billian (ii) Bifurcate in the word as in the English alphabet (in (iii) Bilateral (iv) Bilirubin both forward and backward direc tions} {SSC (Multitasking) 2013] wo arse 201) (b) (iv) (ii) ii) (i) (a) None ne (¢) Two tae : ee @ Troe _— ey oan tee | 11. (i) Ambitious (ii) Ambigngys-——"~ '18:-How many such pairs of letters are there mal 7 in the word ‘SUBSTANCE’, each of fy Ambiguity (wy) na ee cw nots which has as many letters between them ee nee a in the word (in both forward ard (2) Gin,dtv9.0.0) Gi... backward directions) as in the English (©) Gid.C.O.0).) (BG... alphabet? 119s Po) 21 12, In dictionary, which word comes fourth (@) None (b) One (©) Two in arrangement? [SSC (Multitasking) 2013] (@) Three (©) More than three (@) Propense (b) Prophet 19. How many such pairs of letters are there (©) Prong (@) Propine in the word 'STONED', each of which has as many letters between them in the 13. Arrange the following words as per order word (in both forward and backward in the dictionary? [SSC (CPO) 2013) wor Gn both ierward andl ac . rections) as they have between them in Divide: Gi) Division the English alphabetical series? Devine (iv) Divest [Punjab Grameen Bank (Clerk) 2011] \ (v) Direct (a) None (b) Three (c) Two @Mm.a.ac — ©) Mm~o.in, @ (@) One (©) More than three (©) (@.C). (ill) (iv), (v) (A) (iit). (v), (iv), (i), (i) Leanicu vy Canovannier yoursmahboob.wordpress.com 20. How many such pairs of letters are there in word "ENGLISH, each of which has as many letters between its tw between them as ‘englisn alphabet? in the English (@) None (©) One (©) Two (9) Three (@) More than three 21. How many such pairs of letters are there in the word ‘FOREHAND, each of which have as many letters i between thi they have in the English alphabet?" (a) None (©) One (e) Two (@) Three (©) More than three 22. The positions of how many alphabets will remain unchanged if each of the alphabets in the word ‘FORGET" is B. Expert Level Exercise Directions (Q. Nos. 1-5) Study the following arrangement of numbers, letters and symbols carefully and answer the questions Biven below. UUBPS (Clerko 2041] R@29TVAY5©#J1P8QSE3%H %6WALSUZ 1, Four of the following five are alike in a certain way based on their positions in the above arrangement, and so form a group. Which is the one that does not belong to that group? (a) JPO (b) ExQ (© wi% @ ve (1#8 2. Which of the following is the fifth to the right of the nineteenth element from the Tight end? (aP (oe) V Ow @s (€) None of these 3. How many such number are there in the given arrangement, each of which is immediately preceded by a consonant and immediately followed by a symbol? (a) One (0) Two (c) Three (@) Four (e) More than four ‘Scanned by CamScanner Chapter 4+ Alphabet Test 49 arranged in alphabetical order from left to right? [ust (PO) 2011) (a) None (b) One (©) Two (@) Three {(@) More than three 23. Four of the following five are alike in a certain way based on the English alphabetical series and so form a group. Which is the one that does not belong to the group? [Punjab Grameen Bank (Clerk) 2011] (@) BRA (©) FUT (9) oxv (a) CSR (e) JZY 24, In the following list of English alphabets, one alphabet has not been used. Identify the same. [SSC (CGL 2012} XNFAPSRWLTMDEXMGBCX QJLOPVRCQJZOHSGODIPT SMRABEFGNUNE @y ©! ok OR) 4.If the positions of the last eighteen elements in the given arrangement are reversed, which of the following will be the seventeenth from the left end? @e wp (ow ws {€) None of these 5. How many such vowels are there in the given arrangement, each of which is either immediately followed by a symbol or immediately preceded by a symbol? (a) Four (0) One (©) Two (d) Three (©) None of these Directions (Q. Nos. 6-8) in each of the following questions, find out which of the letter-series follows the given rule. . Number of letters skipped in reverse order in between adjacent letters in the series is constant. (@) SQW (b) SPNLJ (c) SPMJG (@) WuTRQ yoursmahboob.wordpress.com sO TN < skipped in between acta decrease in order a) avon PY UNSW NSKCH 2 AGMRV 8. Number of letters skipped in between the adjacent letters in the series is equal a) SUXADF RVZDML eo) HKNGSW 3) RVZDFG Answer with A Base Level Exercise 1. (¢} Alphabetical order 1s Crude, Cruise, Crumb, Crupper, Crusade So, word ‘Crumb’ will come in middle 2. (e) Alphabetical order is Miniature, Minwmalis, Minister), Minisulate, Minonty So. word Ministerial’ wil come in middie. 3. (0) Aipnabetical order is Sentence, Sententious, Sentimentally, Sentimentiaize, Sentine! So. word 'Sentimentally’ will come in middle. 4. (d) Alphabetical order is Frankalmoign, Frankenstein, Frankincense, Fravendienst So. the word ‘Frauendienst’ would be the last, word 5. (c) AS per the Dictionary, the sequence of words is as follows Encounter -+ Encourage -» Encradle —+Encroach 6. (d) The order of words according to the English ictionary is as follows ‘Quality, Question, Quick. Quit ‘So, Quit will appear fourth in English dictionary. 7. (0 Board ~+ Bonus — Bound —» Bunch > wy, 0. i) Scanned by CamScanner How to Crave Test of Reasoning Verbal rections (2. Nos. 9-10) Study the Oe of numbers, symbols and apne eretuly and answer the QUESTIONS that royge’t> {Allahabad Bank cig Ac78FSI9GQ314D+U2@4e™ 9. Which of the following’is the 3rd to tty eft of 18th element from the left? ws 1 (6) None of these 10. How many such symbols are there in above arrangement each of “which immediately followed by a digit? (a8 WE (a) Two (b) Three (©) One (0) Zero (e) Four Explanations {8 (0 As per order in the dictionary arrangement of hy {gyven words is (i) Aquarium-> (iy) Aquatic — (i) Aqueous +) Aquiing 9. (0) AS per the dictionary order is. Preparative, Preponderate, Preposttion, Preposterous. > Gi), Gv). (, (ii) 10. (¢) Correct order will be Bifurcate > Bilateral > Bilirubin > Billain 2 i, Gi, . 11. (¢) As per in dictionary, the sequence is as follow Ambiguity. Ambiguous, Ambitious, Animals, ‘Animation > Gi... ©. 12. (@) Proper arrangement of words is as follows Prong > Propense > Prophet > Propine 13. (d) The correct order is as follow Devine, Direct, Divest, Divide, Division ie, (id, (¥), (), 0. i) 14.0 PREAMBLE So, such pairs are—AE and BE yoursmahboob.wordpress.com ‘So, such pairs are — IN, GH and E1 16. () JOURNEY Such pairs are — JN, UY and Ey. 17.) _>7 STREAM L4 ;" 8 Such pairs are — ST, Gl and NT 18. i— SUBSTANCE L+$t_y ‘Such pairs are — ST, AC and SU. 19.) STONED Ltt td Such paris are — ST, NO and DE B, Expert Level Exercise Les P—So Ease atsi-+% 94v—+@ 1 #55 Hence, except option (e) all others are of the same group. 2. (e) Fifth to the right of the nineteenth element from the right end = Fourteenth element from the right end =Q 3. (a) Only one number 5, which is immediately preceded by a consonant and immediately followed by a symbol. 4. (¢) After rearrangement, R@ 29TVAY5 © #2U BI4W6%H*SESQGP 1) 5. (d) Only three vowels E, | and U are in the arrangement immediately followed by a symbol or immediately preceded by a symbol. Chapter 4- Alphabet Test 51 20. (@) ENGLISH So, such pairs are — EG, El, Gi and LN. 2.0F OR EH AND So, such pairs are — AF and NA FoRG eft Alphabetical Order EF GO RLT So, after arranged the letters alphabetically 'T'is the only one letter which takes same piace. 22. (0) Bib RQ OX V J Zz Y Loar LHLt Ltt 76 ~~ “6 FuT CSR 24, (¢) From the list of English alphabet letter K has not been used. (go through every given options) 695 Pp M Js G 71@s Y D H K 8. (6) R Vv Zz D H L ae oo AcTSFE$I9GQ31AD+U2@4C a STE Hence, in the above arrangements 18th from the left is 2 and 3rd to the left of 2 is D. 10. (6) E7,93.@ 4= Three ee ‘Dcai jeu Dy Camiocal yoursmahboob.wordpress.com Word Formation ‘Word formation’ is advanced level alphabet test. This test is meant to test the ability of the candidate in word building process. In word formation, a main word is given and we have to choose that word, which can or cannot be formed from the main word. Sometimes a set of English letters are given in a jumbled order and the candidates are asked to arrange them in a meaningful order. In some situations we are asked to choose particular letters from a word and arrange them to form a meaningful word, In this chapter, we will deal with two types of questions Type 1 Word Formation Using Letters from a Given Word In this type, three subtypes of questions are asked generally. First a word has been given, followed by four other words, one of which can or cannot be formed by using the main word. Second a word has been given and candidates are asked to make new meaningful words using letters like 1st, 3rd, 5th, 8th, etc., of the given word. Third a word has been given and candidates are asked to form as many meaningful English words as possible from the given word, using each letter only once in each word. Directions (lustrations 1-2) in each of the following questions, a word has been given, followed by four other words, one of which cannot be formed by using the letters of the given word. Find that word. Illustration 1. CONSTRUCTION (a) SUCTION (b) COINS (c) CAUTION Solution (c) CAUTION—All the letters except ‘A’ are present in the main word. Hence, ‘CAUTION’ cannot be formed from the letters of the given word ‘CONSTRUCTION. (d) MOTION Meanicu vy Vaniucanicr yoursmahboob.wordpress.com Illustration 2, NATIONALISATION (a) NOTATION (b) SALINATION (c) INSTALLATION (@) SANITATION. Solution (c) INSTALLATION—All the second ‘U are present in the main wera nPt Hence, ‘INSTALLATION’ cannot be formed t ltrs of he oven werd NATIONALSseea ge Directions (Illustrations 3-4) In each of th questions, @ word has been sien, Tehnet four other words, one of which can be formed by using the letters of the given word. Find that word Mustration 3. MEASUREMENT (a) ASSURE (b) MASTER (© SUMMIT (d) MANTLE Solution (b) MASTER—Al the letters of this word are present in the main word, Hence, ‘MASTER’ can be formed from the letters of the given word "MEASUREMENT: Illustration 4. COMPENSATION (@) TINY (b) Copy () MENTION (¢) MOTIVE Solution (c) MENTION—Ali the letters of this word are present in the main word, Hence, ‘MENTION’ can be formed from the letters of the given word ‘COMPENSATION Chapter 5-Word Formation 53 Mlustration 5. If it 1, possible to make a meaningful word with the Ist, 4th, 7th and Vth letters = fromthe -— word ‘INTERPRETATION’, “hich of the following will be the third letter of that word? If more than one such word can be made, given ’M’ as the answer and if no such word can be formed, give ‘xX’ as the answer. aT (b) R (Mm @) x Solution (c) Since, 1st, 4th, 7th and 11th letters are IE, Rand T, respectively. Hence, two meaningtul words RITE and TIRE can be formed, Illustration 6. How many meaningful English words can be formed by using letters of the word ‘ALEP’? @) One (b) Two (©) Three (d) More than three Solution (c) Such meaningful words are 'PEAL', ‘LEAP’, "PALE Type 2 Word Formation by Unscrambling Letters In this type of questions, a set of English letters is given in a jumbled order. The candidate is required to arrange these letters to form a meaningful word. Always try to place the letter according to the numbers provided in option that will provide you correct answer rather than doing it on the basis of your vocabulary knowledge. Wustration 7. Select the combination of numbers, so that the letters arranged will form a meaningful word. HNRCAB 123456 fa) 2,5,3,4,1,6 (b) 3,5,6,4,1,2 (©) 4,1,5,6,2,3 (d) 6,3,5,2,4,1 Solution (c) Clearly, the given letters, when arranged in the order of ‘6, 3, 5. 2, 4, 1,' form the word "BRANCH Scanned by CamScanner Illustration 8. Rearrange the letters given below to form a meaningful word and select from the given alternatives, the word which is almost opposite in meaning to the word so formed. ARTY,D (a) Dirty (b) Quiet (©) Quick (d) Queek Solution (c) The word is Tardy’ which means ‘Sluggish’, opposite of this word is ‘Quick’. yoursmahboob.wordpress.com Let us Practice A. Base Level Exercise Directions (0. Nos. 1-8) in each of the following Questions, a word has been given, followed by four other words, one of which cannot be formed by using the letters from the given word. Find that word. 1. REASONABLE (a) BONES, (b) BRAIN (c) ARSON (0) NOBLE 2. COMMUNICATION [ssc (Multitasking) 2014] 56 ow to Crack Test of Reasoning « Verbal 4. How many meaningful English words can be formed, starting with S, with the Second, the fourth, the fifth’ and the eighth letters of the word PERISHED, using each letter only once in each word? (to be counted from left) 18s (Clerk) 2011), (a) None (0) One (c) Two (@) Three (€) More than three 5. If it is possible to make a meaningful word with 1st, 5th, 6th and 11th letters of the word ‘COURAGEOUSLY', which of the following will be third letter of that word? Ifno such word can be made, give ‘X' as your answer and if more than one such word can be made, give ’M' as the answer. @o (o) A (G () x eM 6. How many meaningful English words can be formed with the third, the fifth, the seventh and the ninth letters of the word ‘DOWNGRADED’ once in each word? BPS (Clerk) 2012} (@) None (0) One (©) Two (A) Three (e) More than three 7. How many meaningful English words can be made with the letters ‘EPRO' using each letter only once in each word? (a) None (0) One () Two (d) Three (©) More than three Scanned by CamScanner meaningful English | Beebe made with the letters "ERB | using each etter only once in each wo, (@) None (0) One (©) Two (@) Three (@) Move than three 9, How many meaningful English wo, a can be made with the letters “Nyt ing each letter only once in each w, using © UAllhabad Bank (Cer goo? (b) Two (©) None” 7°" One E (©) More than three (O) Three 10. Select the letters that complete the fi, word and begin the second from yh! given alternatives. PLAT (?) ATION ISSCIten0 20, (a) TENT (b) FORM (©) TERR (@ EAU 11, How many meaningful English worg can be made with the letters “ALES using each letter only once in each wordy ISSC 11042 90 (b) One © Two (@) None (©) More than three (@) Three 12. What is the last letter of the word which is formed by rearranging the following letters? CPAOECK (a bird) {EPFO 2011) @eE () K (oP @c (©) None of these 13. The letters of the word ‘NUMKIPP’ are disorder. If they are arranged in proper order, the name of a vegetable is formed, What is the last letter of the word so formed? [CDs 2011) @k () M ()N (a P yoursmahboob.wordpress.com A. Base Level Exercise 1. (6) The word “BRAIN' cannot be formed tr "REASONABLE’ due to absence of letter i 2. (0) UNISON cannot be formed letter 'S' is notin the ven word 3. (0) The word which cannot be formed. form ‘Specitication' is Fainting’ due to absence of the letter ‘G" and only one 'N’ is present in the given word 4. (0) Using the letters of the given word, the word LIZARD cannot be made because D is not there in the given word. 5. (a) From the word ‘CONCEPTUALISATION, the word which cannot be formed is STATUS as it contains only one ‘S, 6. (d) TALENT word cannot be formed using the letters of the given words because letter E is not present in the given word. = (2) The word ‘Secure’ cannot be formed from ‘Courageous’ because only one ‘e' is present in the given word, 8. (d) ‘PORTICO’ word cannot be formed from ‘PORTFOLIO’ due to the absence of letter ‘C' (€) From the given word, “MEDICO’ is the only word Which can be formed, 10. (0) ‘PARROT’ can be formed trom PREPARATION’. 11. (d) ‘DAIRY’ word can be formed from the word "DICTIONARY 12, (b) ‘LIMITED’ is the only word which can be formed by using the letters of given word 3. Expert Level Exercise 1. (6) From letters O, |, T, N, only one meaningful word ‘INTO’ can be formed and last letter of this word is O. 2. (c) The second, fifth, tenth and tweltth letters of the: word METROPOLITAN are E, O, T and N, respectively. The words formed are NOTE and TONE. 3. (c) From letters T, E, C, N, only one meaningful word ‘CENT’ can be formed and third letter of this word is N. 4. (b) Second, fourth, fifth and eighth letters of the word PERISHED ->€, |, S, D Meaningful English word — SIDE 5.1) 18. C, A, G and L no meanin;tul word can be iormed 6.(0)D O jw) N [6] R[A] D[E] 0 Scanned by CamScanner Answer with Explanations 13. (¢) ‘OPINION’ can be formed from ‘COMPANIONSHIP 419283951746 vedgdggag PLAINTIFF 15, (0) MIRACLE «—» (6, 3. 4, 1, 7, 2, 8) 17. (@) STARVE + (4, 5.2, 3, 1, 6) 18. (6) HERMIT <> (4, 1, 6,2, 3, 5) 19. (d) Correct order of given letters is COMPLEX (5). 2). 3). 6). (4.7.0) 2.199 8 1237612 beebudgars HYPOTHECAT 21. (0) NATURE <> (4, 3, 2,5, 1, 6) 22. (0) The same after meaning of the given is INCREASE’ because after arrangement of given ‘word is ‘EXPAND’ 23, (0) The word is ‘DEARTH’ meaning ‘SCARCITY’. 24, (9 Arrangement of word is ‘BEHIND’ and the similar word is ‘BACK’ 25. (a) The word is ‘GARNISH’ meaning ‘DECORATE’. 13945 44 ' oes ze ‘Only one meaningful English word WAGE can be formed by using the letters WGA and E. (©) Two meaningful word ‘ROPE’ and ‘REPO’ can be formed by using letters from 'EPRO’. 8. (b) Only one meaningful word "RUDE’ can be formed by using letters from "ERDU' 9. (c) Only one meaningful English word can be formed with NWT! is TWIN. 10. (b) ‘FORM’ completes the first word as PLATFORM and begin the second, 11. (d) Only three words can be formed PEAL, PALE, LEAP. 12. (b) The name of the bird is PEACOCK and the last letter is K. (PEACOCK) 13. () The name ot that vegetable would be PUMPKIN’ and last letter of the word is N. = yoursmahboob.wordpress.com Sitting Arrangement rsons on by Sitting arrangement is sequential arrangement of objects pe! Asis of predefined conditions. In this chapter, we deal with the questions based on sitting arrangemeny We are given few objects and then we are asked to arrange them based on th, information provided. In other questions, we will also deal wit! artangemeni, in different shapes like square/pentagon/hexagon. Types of Questions There are two types of questions which are generally asked in Various competitive exams Type 1 Arrangement in a Line or Others Here, we deal with questions, where arrangement is Tequired to be done in a line or any others format in a straight line. Before solving this type of questions, it is necessary to know the following facts * When A, 8, Cand D facing at South direction and P,Q, Rand S facing at lirecti in a line, then positions at their right and left will be emai Nowh rection Fit Ap Dla ie North Southward facing Scanned by CamScanner yoursmahboob.wordpress.com Chapter 6-Sitting Arrangement 59 Ilustration 1. In shop, the items are arranged Mlustration 2. How many persons are there to ina shelf consisting of six rows. Biseuite wre ‘the right of D? kept above the tins of chocolates but below {a) One (b) Two (© Three the rows of packets of chips, cakes are atthe (¢) Four (e) None of these bottom and the bottles of peppermints ai , a below the chocolates. The topmost Tow keg Hiustratton 3. Which of the following is sitting the display of jam botth to the left of D? the bottles of Peppermint mene! ae @F wc oe place from the top. @A (@) None ofthese (a) 2nd (31d) thd) Sth Illustration 4. Who is at the immediate left Solution (2) Jam bottles is at the top. Now, erran of C? ‘them according to the question, we have ai (a A Chips (Ghecoiates WE I (©) Either Eor A Biscuits Peppermints ] (d) Cannot be determined (e) None of the above Crovoltes| Cakes Mlustration 5, Who is at the right end? re @A Jam bates ee WE ore (d) Cannot be determined (e) None of the above Biscuits pe Roms Solutions (\Wustrations 2-5) The given figure, shows the yrrect | yement of six friends. Comes ] correct aang ' Peppermints: FE op A ce By « Cakes Siting arangement 8 Directions (ilustations 2-5) Stugy the following 9, ee information caretuly and answer the question, 2) Tete are four persons to the ight of DA, C, given below. 3.(a) Clearly, Fis sitting to the lett of D. Six friends A, B,C, D, E and F are sitting ina 4.) It'is clear from the diagram that A is at the row facing towards North. C is sitting immediate lett of C. between A and E. D is not at the end. B is 5. (6) Its clear from the diagram that B is at the right sitting at immediate right of E. F is not at the end, right end but D is sitting at 3rd left of E. Type 2 Arrangement Around a Circle / Square / Rectangle etc. In such questions, we are given some clues regarding the arrangement. We have to apply these clues for sitting arrangement and using these informations, we have to find the solutions to questions associated with them. Here, we also deal with square/pentagon/hexagonal /arrangement in this section. Scanned by CamScanner 50 yenrcomtekbook:werdpress.com Belore solving type 2 questions, it is necessary to know the following facts. © For Circular Arrangement {mn this arrangement, suppose some persons are Directions (itustrations 6 P,Q, R,S, 7, U, Vand sitting around a circle and they facing the centre. coffee. Font ( Movement towards the left is also called clockwise rotation. (i) Movement towards the right is also called anti-clockwise rotation. For Rectangular/Square Arrangement AC (facing each other) B 4+ D (facing each other) * For Pentagonal Arrangement A E 8 Fon Let o c * For Hexagonal Arrangement A Right, AeDd (facing each other) 5 <> E (facing each other) CF (facing each other Scanned by CamScanner 10) carefuly ‘oy ‘based on it, 7d answer the Ware eight fr are sitting around around “iS fiends = Read the # table and 7 ( Pis siting between U and v, (i) Qs siting between W and +. Lil) Ris sitting to the third left of v, Gv) W is sitting third right of §, | Mlustration 6. What is the position of y | respect to S? } (a) Immediate left j (b) Immediate right | (©) Third to the left (4) Fourth to the right Illustration 7. How many per between Rand s7” PETSONS are Siting (a) 2 (b) 3 4 @s ‘Dlustration 8. What is the position of T with respect to W? (3 mrmedite ef () immediate ry (©) Third tothe left (d) secon to theleg, Illustration 9. Which of the following is tue (a) Vis sitting immediate right of R " (b} Uis sitting immediate left of p {©) There is only one friend between W and (d) All of the above Mlustration 10. Which of the following is fase) {2} There are two friends between P and we (b) There is no one between W and a (©) Pis immediate right of v (d) Sisto the second left of Solutions (itustrations 6-10) Adjoining figure s Sorreet sitting arrangement of al eight friends siting around @ round table ? 6. (@) Itis clear from the diagram that V is at immediale| left of s, yoursmahboob.wordpress.com Chapter 6+ Sitting Arrangement 61 7. (0) Clearly, three members are sit ing between R and S, 8. (@) Clearly, Tis sitting to the second left of W. 9. () Uis siting immediately to the right of R. tne) Uis sitting immediately to the lett of P. (tr There is only Q between W and T. es 10. (c) There are two friends between P and W. (vue There is no one between W and Q fae) Pis immediately to the right of V. ree) Sis to the left of Q leaving one, ~ Direction (illustration 11) Read the following information carefully and answer the questions based on it. ix persons M, N, O, P,Q and R are playi ena Rest ven , Pr playing cards sitting in a circle facing the centre. R is sitting between M and N and Q is siting between © and P. Ps sitting at immediate right of M. Mlustration 11, Who is sitting immediate left of 0? (@) R (om wa @P (©) Cannot be determined Solution Adjoining figure shows the correct arrangement of siting of these six persons. N R Oo Mw Q & 11. (c) Clearly, Q is sitting immediately at the left of O. Let us Practice A. Base Level Exercise 1, There are five different houses, Ato E, in Directions (Q. Nos. 3-5) Read the following a row. Ais to the right of B and Eis to the information carefully and answer the questions left of C and right of A, B is to the right of given below it TSSC (Constable) 2012] D. Which of the houses isin the middle? —_, 8, C, D, E, F and Gare playing cards {1B ACIO) 2013} circle. @A OE ue | jc @D (i) F is 2nd to the right of G. 2. A, B, C, D, E and F are sitting in a row. iD Isnetghbour of F Dat natiof E, "B’ and “F" are in the centre and A’ and (iii) E is neighbour of C, Who is 4th to the right of G. ‘Bare at the ends. ‘C'issittingontheleft =, G. of ‘A’. Then who is sitting on the right of __¥) Dis between E and A. ‘Bz [SSC (Multitasking) 2014] 3, Wha is fourth to the left of G? @A (b)D (@o @E OF (bE wc (¢) Cannot be determined Scanned by CamScanner yoursmahboob.wordpress.com 52 How to Crack Test of Reasoning « Verbal 4, Who is the left of G? @a mc oes (A) None of these 5. Who are the neighbours of F? (@) E and. c (b) F and B (c) AandB (d) CandB Directions (Q. Nos. 6-9) Read the information carefully and answer the questions based on it Vijaya Bank (Clerko 2019] ix persons are sitting in a circle. A is facing to B- B is to the right of E and left of C. C is to the left of D. F is to the right of A. Now, D exchanges his seat with F and E with B. 6. Who will be sitting to the left of D? @e OF Ee (A ~@c 7. Who will be sitting to the left of C? @e OF A 8 (dD 8. Who will be sitting opposite of A? @e oF (0 8 @c 9. Who will be sitting opposite to C? @e oo (8 (A @F 10. At a birthday party, 5 friends are sitting ina row. 'M’ is to the left of ‘O' and to the right of ‘P’. ‘S’ is sitting to the right of 'T’, but to the left of ‘P’. Who is sitting in the middle? [SSC (10+2) 2013] (@M (b) 0 (o)P @s |. Ina gathering seven members are sitting in a row. ‘C’ is sitting left to ‘B’ but on the right to ‘D’. ‘A’ is sitting right to ‘B’, ‘F’ is sitting right to ‘E’ but left to 'D’. ‘H’ is sitting left to ‘E’, Find the person sitting in the middle. [SSC (1042) 2013] fac OE (o)D QF Scanned by CamScanner I i 1d T are sith 12. Five friends P, Q, R, S an ting, ing North. Here S is betweg' 2 Tow tar Q is to the immediate lef, at] Pst the Gimediate lett of T. Who the middle? ISSC (Mulitasing 8 @s oT " a @R i Nos. 13-17) Study the infor Directions 10d answer the questions that fj 5, T, U, V, W, X, ¥ and Z are sitting aroung , Circle area, with equal distance amongst each other but not necessarily in the same order, Only two people face the centre and the rey face outide (i.e., in a direction opposite to the centre). Y sits second to left of W. S$ sits second to left oy Y. Only one person sits between S and Z. T sx, to immediate right of S. T is not an immediate neighbour of Y. V is not an immediate neighbour of Y. Both the immediate neighbours of X face the centre. 13. Who is sitting te immediate right of z7 @y ov ©T (dX (ew 14. Which of the following is true regarding Uas per the given sitting arrangement (@) X sits second to left of U (©) Only three people sit between U and (©) Zis one of the immediate neighbours of U (d) U faces the centre (@) S sits to immediate left of U 15. What is T’s position with respect of Y? (@) Second to the right (6) Second to the left () Fifth to the left (d) Fourth to the right (€) Third to the left 16. Which of the following groups represents the immediate neighbours of X? (a) WY (b) WwW (c) TZ () vz (e) SU yoursmahboob.wordpress.com * are alike ina aiven seating jee orm a qroup. Which ne that does not belong to that qroup? 2 aT a wx wy Directions (Q. Nox 18 20) Siu : oases ie gagneer nents six people-K. LM... O and P fi Ginerent floors of a buikhing not nececarly ie the same order. The lower most floor of the building 1s numbered 1, the one above that numbered 2 and so on tll the top most floor is numbered 6. L lives on an even numbered floor L_ lives immediately below K’s floor and immediately above M's floor. P- lives immediately above N’s floor. P lives on an even numbered floor. O does not live on floor number 4 18. Four of the following five are alike in a certain way based on the given arrangement and hence form a group. Which of the following does not belong to that group? (a) MN (©) OL (©) kM wr (@) PK 19. Who amongst the following lives on floor number 2? (a) K (o) P (Le (QM (0 10. On which floor does N live? (a)4 (0) 3 os 1 (2 1. Five boys A, B, C, D and E are standing row, Dis on the right of E. Bis on the left of E, but on the right of A, Bis on the left of C, who is standing on the extreme right. Who is standing in the middle? [SSC (10+2) 2013] (a8 (o) C (0 @e 2. Six friends A, B, C, D, E and Fare sitting in a row facing towards North. C is sitting between A and E, D is not at the end, Bis sitting at immediate right of E, Chapter 6+ Sitting Arrangement 6S F is not at the right end, but D is sitting at 3rd left of E, Which of the following is sitting to the left of D? A DF we Directions (Q. Nos. 23-28) Study the following information carefully and answer the given questions. {wars Fo) 2092) (i A,B,C, D, E, F, Gand H are sitting around a circle facing the centre but not necessarily in the same order. (ii) B sits second to left of H's husband. No female is an immediate neighbour of B. (iii) D’s daughter sits second to right of F who is the sister of G. F is not an immediate neighbour of H’s husband. (iv) Only one person sits between A and F. A is the father of G. H’s brother D, sits to the immediate left of H’s mother. Only one person sits between H’s mother and E. () Only one person sits between H and G. G is the mother of C. G is not an immediate neighbour of E. 23. What is position of A with respect to his mother-in-law? (a) Immediate left (0) Third to the right (c) Third to the left (d) Second to the right (@) Fourth to the left 24. Who amongst the following is D's daughter? (a) B (o) C OG (eH 25. What is the position of A with respect to his grandchild? (a) Immediate right (b) Third to the right (©) Third to the left (@) Second to the left (e) Fourth to the left wc WE 26. How many people sit between G and her uncle? (a) One (b) Two. (c) Three (0) Four (@) More than four yoursmahboob.wordpress.com 64 How to Crack Test of Reasoning « Verbal 27. Four of the following five are alike in a certain way based on the given information and so form a group. Which is the one that does not belong to that group? (ar mc Ee (OH @G 28. Which of the following is true with Tespect to the given sitting arrangement? (@) Cis the cousin of € (0) H and H's husband are immediate neighbours of each other (c) No female is an immediate neighbour of C (0) H sits third to left of her daughter (@) Bis the mother of H Directions (Q. Nos. 29-31) These questions are based ‘on the information given below. Six girls are sitting in a circle. Sonia is sitting opposite to Radhika. Poonam is sitting at right of Radhika but left of Deepti. Monika is sitting at left of Radhika. Kamini is sitting at right of Sonia and left of Monika. Now, Deepti and Kamini, Monika and Radhika mutually exchange their positions. 29. Who will be sitting opposite to Sonia? (a) Radhika (b) Monika (c) Kamini (q) Deep —_(e) None of these 30. Who will be sitting at left of Kamini? (a) Poonam — (b) Deepti —_(c) Radhika (0) Sonia _(@) None of these B. Expert Level Exercise Directions (Q. Nos. 1-2) These questions are based ‘on the information given below. A group of seven singers, facing the audience, are standing in a line on the stage as follows (i) isto the right of C. (i) F is standing beside G. (iii) B is to the left of F. (iv) E isto the left of A, () Cand B have one person between them. (Vi) A and D have one person between them. Scanned by CamScanner 31, Who will be sitting at Jett on . (b) Moni (a) Sonia (@ Rechika __(d) Poonam (@) Cannot be determined rections (Q. Nos. 32-35) These questions are ° ‘on the information given below. ake, Sh 20, Q,R, S, T and U are sitting jn?" cle facing one another front to front. p'2 Sitting in front of Q. Q is sitting to the right oft and left of R. P is sitting to the left of U and rig of S. 32. Who is sitting opposite to R? Six persons P, (a P ) a os @T eu 33. Who is sitting opposite to S? (@u (b) T (QR (@) Cannot be determined (@) None of the above 34. Who is sitting between P and R? @s (b) T @u @a (e) None of these 35. If the positions of P and R ay interchanged, who will be siting between S and U? (@P oe ()Q @)T (e) Cannot be determined 1. Who is on the extreme right? (aD (b) F @G @e (e) None of these 2. If we start counting from the left, on which number is C? (a) 1st (c) 3rd (e) None of these (b) 2nd (@) 5th ee * yoursmahboob. Woudpe ess, 6 Qe 65 Directions (Q. Nos. 3.4) Read the follow ing, merpabon carefuly and answer the questions {i Six flats on a floor in two and South ae allotedtoP, Q. 8, Sy eNO (iv) Rnextto U, gets a South fae 3. Whose flat is between @ and $7 (aT ®)uU @AR @P (@) Data inadequate 4. The flats of which of the other pair than SU; is diagonally opposite to each other? (@ Pr (©) ap (© QA (@) Ts (©) None of these. Directions (Q Nos. 5-10) study the following information to answer the given questions. UBPS PO) 2012) Ten people are sitting in two parallel tows Containing five people each, in such a way that there is an equal ‘distance between adjacent persons. In row 1, P, Q, R, S and T are seated and all of them are facing South. In row 2, , B, C.D and E are seated and all of them are facing North, Therefore, in the given seating arrangement each member seated in a row faces another member of the other row. D sits third to the left of A. P faces immediate neighbour of D, R sits second to the right of P. Only one person sits between Q and S, B and E are immediate neighbours of each other. E does not face P and Q. 5. How many persons are seated between QandT? (a) None (b) One (©) Two (d) Three (©) Cannot be determined 6. Four of the following five are alike in a certain way and thus form a group. Which is the one that does not belong to that group? @R rT (os @A @c 7. Who amongst the following represent the People sitting exactly in the middle of the rows? @Pe ©) 5,0 SA OAR @PB 8. Which of the following is true regarding B? (@) Aand C are immediate neighbours of B (©) 8 sits at one of the extreme ends of the line (©) Q faces B (@)T is an immediate neighbour of the person facing B (€) D sits to the immediate left of B 9. Four of the following five are alike in a certain way and thus from a group. Which is the one that does not belong to that group? @TE (b) O-C Oss (RA PD 10. Who amongst the following faces S? @A 8 ce @od @eE Directions (Q. Nos. 11-15) Study the following information to answer the given questions. {SBI (Clee 2012), (i) Eight persons E, F, G, H, 1,J, Kand L are seated around a square tabletwo on each side. Gi) There are three lady members and they are Not seated next to each other. (il) J is between L and F. (iv) Gis between | and F. (v) H, a lady member, is second to the left of J. (vi) F, a male member, is seated opposite to E, a lady member. (vii) There is a lady member between F and |. 11. Who among the following are the three lady members? fa) E, Handy (©) GH and J (©) None of these (0) E,GandJ () H, EandG 12, Which of the following is true about J? (@) Jis a male member (0) J is a female member (©) Sex of J cannot be determined (@) Position of J cannot be determined (@) All of the above yoursmahboob.wordpress.com 5 tow to Crack Test of Reasoning « Verbal 13. Who among the following is seated between E and H? (1 oJ (oF kK (e) None of these 14. Who among the following is to the immediate left of F? G ws (I @ Kk (@) None of these 15. How many persons are seat, red bey, and F? Seay (@) One (b) Two (©) Three (@) Cannot be determined (©) None of the above Directions (a. Nos. 16-20) Read the information given below to answer the questions. A,B, C, D, E, F, Gand H want to have a dinner on seating arrangements. @ A will sit beside C (iv) F will sit beside H (vi) G will sit beside B 16. Which of the following is wrong? (2) A will be to the immediate right of © (©) D will be to the immediate left of B (c) E will be to the immediate right of A (©) F will be to the immediate left of D 17. Which of the following is correct? (@) B will be to the immediate left of D (0) H will be to the immediate right of A (©) Cwill be to the immediate right of F (2) B will be to the immediate left of H (ii) H will sit besi ~— 3 round table and they have worked out the fo (v) E will sit beside G (viii) B will sit beside D lowin, [etary (ii) C will sit beside E (vi) D will sit beside F e ide A " ° A and F will become neighbours if (2) B agrees to change her sitting position (©) C agrees to change her sitting position (©) G agrees to change her siting postion (@) H agrees to change her sitting position 19. During sitting (@) A will be directly facing C (©) B will be directly facing G (6) A.will be directly facing B (2) B will be directly facing D 18. 20. H will be sitting between (0) A and F (@) E and g (@) CandB (c) Dand G Eight people-s, R, N, LM, T, O and P are sitting in a circle facing the centre. All eight bel Professions-reporter, doctor, Cricketer, teacher, accountant, shopkeeper, painter oom are not necessarily seated in the mentioned order. M is sitting third to the left of O. The doctor is to the immediate right of M and M is not rey is in ia Porter. R is sitting fourth to th pee Pis an immediate neighbour of M, T is a teacher and is sitting thin to the right a ee ing econ fot the an of the teacher, The painter is sitting second to ti a /een , is sit is siting thin on pA wes and P. The Accountant is sitting second to the 21. Who amon, Is the fc ing is is bis 1g following is a Teporter? 22. What is S's position with respect to Ri OL (@) Third to the right ON (b) Second to the right OR (©) Third to the left (©) None of the above Sameayc y CamScanner (9) Second to the lett (©) Fourth to the right yoursmahboob.word 23, How many people are sitting between P and N when counted in an anti-clockwise direction from N? (@) One (©) Two. (©) Three (@) Four {€) None of these 24, Four of the following five pairs are alike in a certain way based on their positions in the above arrangement and so form a group. Which of the following does not belong to the group? (@) Teacher-Painter —(b) Supervisor-Shopkeeper *(©) Cricketer-Reporter (d) Doctor-Accountant (6) Shopkeeper-Doctor 25. Which one of the following statements is false according to the above mentioned arrangement? (2) Nis to the immediate right of the supervisor (©) The cricketer is third to the right of the shopkeeper : (©) The doctor is sitting exactly between the supervisor and the accountant (@) Lis neither a teacher nor a supervisor (€) There are only three people between S and N 26. A, P, R, X, S and Z six persons sitting in a row. S and Z are at the centre and A and Pare at the ends. R is sitting on the left of A. Then, who is sitting on the right of P? 188C (CPO) 2003], @A Ss Zz Directions (Q. Nos. 27-29) Study the information carefully and answer the questions given below. (i) Eleven students A, B, C, D, E, F,G, H, I,J and K are sitting in a row of the class facing the teacher. (ii) D, Who is to the immediate left of F, is second to the right of C. (iii) A, who is second to the right of E, Who is at one of the ends. (iv) J is the immediate neighbour of A and B and third to the left of G. (¥) Histo the immediate left of D and third to the x right of |. 27, Who is sitting in the middle of the row? @c (I (9B @G (€) None of these Scanned by CamScanner ress.cOom Chapter’6 - Sitting Arrangement. 67 28. Which of the following groups of friends is sitting to the right of G? (@) IBUA (b) ICHDF (c) CHOF (@) CHDE (©) None of these 29. In the above sitting arrangement, which of the following statements is superfluous? (@) Only (i) (b) Only (i) (©) Only (il) (d) None is supertiuous (@) Cannot be determined Directions (Q. Nos. 30-34) Read the following informations and answer the questions given below. (i) A,B, C, D, E, F, G and H are sitting in a row facing North. A’is-fourth to the right of E. H is fourth to tt of D. ii (i (iv) C and F, which aré\pot at the ends, are neighbours of B and E, Fespectively. (v) H is next to the leff of A and A is the neighbour of B. 30. What is the position of F? (a) Next to thé right of E (b) Next to the right of G (©) Sixth to the right of D (@) Between G and H {€) None of the above 31. Which of the following statements is not true? (@) Gis the neighbour of H and F (b) B is next to the right of A (©) Eis at left end (@) Dis next to the right of B (©) None of the above 32. Who is/are the neighbour(s) of D? (@) F alone (b) C alone (©) Band (8) Cannot be determined yoursmahboob.wordpress.com 68 How to Crack Test of Reasoning + Verbal 33. Which of the following statements is not true? (@) His second to the right of F (0) Eis fourth to the left of A (©) Dis fourth to the right of H (@) Ais third to the let of D (e) All are true 34, Who are sitting at the ends? (@ EandD (0) FandD (© Gand B (@) Cannot be determined (@) None of the above Answer with Explanations A Base Level Exercise 1. (@) Data is arranged as DBAEC So, the house in the middle is A. 2. (6) According to the question, sitting arrangement is at follow BD E/F FECA Clearly, D it sitting on the right of B. Solutions (Q. Nos. 3-5) From the given information, sitting ‘rangement ofl the persons is as follow 4, E Gl Ic 8 e 3. (¢) Person C is fourth to the left of G. 4. (a) Person A is to the left of G. 5. () C and B are the neighbours of F. Solutions (Q. Nos. 6-9) From the given information, sitting arrangement of all the persons is as follows A A F D D After r interchange e ic a Initial position New position 6. (@) itis clear from the figure that A is to the left of D. 7. (0) E is siting to the lett of C Scanned by CamScanner B. (a) Eis siting opposite to A. 9. (0) D is siting opposite to C. 10. ! t t { { N TS P MO LL. er Right” So, P is sitting in the middle. 11. (6) According to the question, arranging the seven members in a row, HEFOCBA Hence, D is sitting in the middle. 12. (a) According to the questions, sitting arr of al the five frends is as follows "oe" PTSQR Hence, S is the middle. Solutions (©. Nos. 13-17) From the given infonmat siting arrangement of al the persons is as talon Y U s T 13, (6) 14. (a) 15. (e) 16. (a) 17.6 yoursmanbood. wor Chapter 6-Sitting Arrangement 6S seaten,1 1820 Arnone fe so N- = Ma,= Fone on ° 23. (c) immeciato ot sn = 24, (@) Bis the D's daughter. ma L 25. (¢ Third to the let as, ™ 26. (2) One. me P 27. () Except H, all the persons have one male and tat N fornala sting atthe left and right 18. (¢) 19. (0) 28. () H and husband ar ieredate neighbours ect ne Solutions (Q. Nos. 29-31) We have the order of siting of 21. (@) The sequence of boys in a queue is ts as given inthe figure A*B4ESD 40 Radhika —_sonika Gen ec) (right end) Poonam| Kamini "2 Poonam| So Eb in he mide deep Yona kame —Soni8 22. (t) According tothe quest COE thd Fie easton, arrangement ofA.8, 29, (6) Monika wil be at the opposite to Soria. FDACEB 30, (a) Poonam willbe siting just lett of Kamin. Thus, F is sitting to the left of D. 31. (a) Sonia wil be sitting just left of Deepii 2-38) Fi exact Solutions (0. Nos. 23:28) From the given information, — SOtmone, (0 Now. ess) Howe, crows ne ‘siting arrangement of all he persons is as poston inthe sting rangement 1 is 32. (@) Uis sitting opposite to R. 33. (b) Tis sitting opposite to S. 34. (a) S is siting between P and R. 35. (b) R will be sitting between S and U because the position of P and R has been changed. O's daughter B. Expert Level Exercise Solutions (Q. Nos. 1-2) Given figure shows the comect Solutions (Q. Nos. 3-4) Given figure shows the correct ‘positions of singers in the line. positions of persons. x Nee ee ara — AC OD BF G South facing» »——»——» uv oR Pow £ 1. (¢ Its clear from the diagram that singer Gis atthe North facing-» «+ extreme right of the line. qT s 20 {ts cee fom the diagram that C is the thd 3 4) Carly, Ts at is ows Scanned by CamScanner yoursmahboob. 70 How to Crack Test of Reasoning - 4. (0) As gven in statement (i), Sand U_ are diaganaty opposte, Srey rom he tures we can say P and Q are also diagonal opposite. ‘Solutions (Q. Nos. 5-10) From the given information, Sting arrangement of all the persons is as fofow Row 1. facing South: +R Q PS T Row 2, facingNoth:+C D BE A 5 610) 70) Be) 9 = 10. (@) ‘Solutions (Q. Nos. 11-16) From the given information, ‘Sting arangement oa re persons ses flow ©® © © © ©) OO Note + = Male; - = Female 11. @) It is clear trom the figure that members are H, E and G, 12. (0) It is clear that J is a male member. 13. (0) K is sitting between E and H. 14, (0) Jis siting immediately left to F, 15. (¢) There are three persons seated between K and F on either side. Solutions (2. Nos. 16-20) Information is aranged as follows the three lady 16. (©) E is not to immediate right of A. 7. (6) H will be on immediate right of A, 8. (a) A and F will become neighbor on, i agrees In, if H Change her sitting positon “te Scanned by CamScanner wordpress.com Verbal 19. (9 Aw be dot facing 8 20, (ai sting boon Ae F Wore (0, Now 21-28) From Be given Solute arargoment of al he porene oa, (Supervisor) ‘ley M (Doctor) N L (Accountany) (Shopkeeper) Fr P (Painter) | (Reporter) 0 S (Cricketer) (eacher) 21. 22. (a) 23. | 24. (e) 25. (0) 26. (¢) Given figure shows the Correct Positons oy. | persons | PX S ZR A Hence, itis clear that X is on the right of p Solutions (Q. Nos. 27-29) On the basis op mn informations given in the question, we have - arrangement of standing as per the figure betey! exkASBTG&ERER 27. (6 tis clear trom the figure drawn on the basi y the information that student I is siting in ya middle of the row. 28. (c) CHDF are sitting to the right of G. 29. (d) All the information is required to Position of all students in the row. Solutions (Q. Nos. 30-34) The order of sitting of all the eight persons which satisfies all the condltions wil be as in the figure. eronewtt HA BC oD the above order that F is next io know the 30. (a) It is clear from the right of E. 31. (@) Statement right of B. 32, (b) Because D is to the only one neighbour C. (0) is wrong because C is next tothe extreme right hence, has 33. (e) All the statements are correct. 34. (0) E and D are sitting at the extreme ends. yoursmahboob.wordpress.com Direction Sense Test Direction is a measurement of position of one thing with respect to another thing and displacement is the measurement of distance between the starting and the final point. _In this chapter, we deal with the questions in which a successive follow-up of directions is formulated and the candidate is required to ascertain the final direction or the distance between two points. This test is designed to judge the candidate's ability to trace and follow the logical path correctly and sense the direction correctly. Main Directions There are four main directions viz. East, West, North and South. Sunrises in the East. Just opposite of East is West and South is in the opposite to North. Left {Ri> Right Right Left Sun set} Left Right | Sunrise Right {Sou}-+ Let Main Directions Abbreviations for these directions are E (East), W (West), N (North) and S (South). Vuaiiied Uy Vena: = yoursmahboob.wordpress.com 72 How to Crack Test of Reasoning « Verbal Cardinal Directions A direction between two main directions is called cardinal direction. Clearly, there 4 four cardinal directions. They are NE (North-East), NW (South-East) and SW Shortest Distance For finding shortest distance, i, . (North-West), SE fragrant, Students are advised to uses _ necessary to know Pythagores theorem, © © of sensing directions. A N nw NE a 8 ¢ w < — AB = Perpendicular sw ‘se BC = Base and AC = Hypotenuse 7 Hence, for shortest distance between 4 Angle formed between two main and C directions is 90° and angle formed between a = (ABP Be cardinal direction and main direction is 45°. jen AC = JAB’ +BC Angle of Movement For solving questions based on angle of movement, itis necessary to know the rotations which are given below ()) Movement at the right direction is called the clockwise movement. 360° 315 45) 270" Clockwise 225 135) 180 (ii) Movement cowards the left is called anti- clockwise movement. 360° o 45, 315° Anti-clockwise| 90° 270° 136° 225° 180° Scanned by CamScanner - yoursmahboob.wordpress.com Chapter 7+Direction Sense Test 73 Types of Questions There are three types of questions which are generally asked in various competitive examination Type 1 Based on Final Direction In this type of questions, we have to ascertain the final direction with respect to starting point or the directional relations between two points/persons/things are used in questions. Mlustration 1. From her house, Avantika went Tustration 2. Of the five villages P,Q, R,S and 15 km to the North. Then, she turned West and covered 10 km. Then, she turned South and covered 5 km. Finally turning to East, she T situated close to each other, P is to the West of Q Ris to the South of P, Tis to the North of Q and S is to the East of T. Then, in which covered 10 km. In which direction is she from direction R is with respect to S? her house? (a) North-West (@) East (b) West (b) South-East (© North (d) South. (c) South-West Solution (c) Movements of Avantika are shown in the (d) Data inadequate ‘given figure Solution (c) Locations of the given villages can be R__19km y shown according to the figure below N 5km wots I NW NE 15km | € TOK 8 | “ P k sw SE s Clearly, Avantika's final position is T which is in the North of her house. Clearly, R lies to the South-West of S. Type 2 Based on Displacement In this type of questions, we deal with distance between two points/persons/things, Mlustration 3. Vinay moves __towards final distance between starting and final point or Solution (c) All the movements of Vinay are shown in the South-East, a distance of 7 m, then he moves towards West and travels a distance of 14 m. From there, he moves towards North-West a distance of 7 m and finally he moves a distance of 4 m towards East and stands at Point. How far is the starting point from where he is standing now? (a) 3m (b) 4m (0 10m @ 11m Scanned by CamScanner figure below m_10m_ ating point AB tom 4 N RI Nw os , Q 4m Pp 8 Vinay’s distance from the starting point (T) =(14-4)=10m yoursmahboob.wordpress.com = 74 How to Crack Test of Reasoning - Verbal Mlustration 4. A person starts from a point A and travels 3 km eastwards to B and then tu \ travels thrice that distance to reach C. ms eh a He again turns left and travels five times the distance he covered between A and B and reach, | ‘SaAectian Dh. Th-aruitoer anne bebe en aE OMSL AOS WNDU MNCEEN “es (a) 12 km. (b) 15 km (©) 16km (d) 18 km hown in the figure below Solution (0) The movements of the person are as s! ote c Ask According to the question. BC = (3x 3)=9 km CD = (3 x 5)= 15k OD = (15~ 3)= 12m Now. required distance, _ AD = VOD? + OA? AD = V12? + 9 = (144+ BF = 1225 km = 15 km Type 3 Based on Direction and Displacement In this type of questions, we deal with the final distance and direction between starting and final point of any person/objecl/thing. Mlustration 5. Shyam goes to 5 km in the North from his school. Now, turning to the left, he goes 1 10 km and again turn to left and goes to 5 km. How far he is from his school and which direction? (a) 10 km, South from schoo! {b) 10 km, North from school (d) 10 km, East from school (6) 10 km, West from school Solution (c) The movernent of ‘Shyam is as follow 10km 5km is km W- = A 2) school Let point A is the starting point ie., school of Shyam Pont D is the ending point. From figure, AB = CD = 5km and AD = BC = 10km So, Shyam is 10 km far away from his school and in West direction from school Scanned by CamScanner yoursmahboob.wordpress.com Let us Practice A. Base Level Exercise 1. Aman is facing West. He t ° Asekmine ng est He tus 4° in the 180° in the same direction and then 270° in the anti-clockwise direction. Which direction is he facing now? IMAT 2013, RRB (CIC 2005), (©) NorthWest (A) South-West 2. A direction pole was situated on the ro crossing. Due to an accident, the oa tumed in such a manner that the pointer which was showing East, started showing South. Sita, a traveller went to the wrong direction thinking it to be West. In what direction actually she was travelling? [SSC (CG) 20131 (@) North (b) West (0) East (¢) South 3.A policeman left his police post and proceeded 4 km South on hearing a loud sound from point A.On reaching the place, he heard another sound and proceeded 4 km to his left to the point B. From B, he proceeded left to reach another place C, 4km away. In which direction, he has to go to reach his police post? (@) North (©) East (@) South (©) West (b) South (d) West 4.A and B start walking in opposite directions. A covers 3 km and B covers 4 km. Then, A turns right and walks 4 km while B tums left and walks 3 km. How far is each from the starting point? 18SC (Steno) 2013] (o) 8 km. (d) 4 km. 5. A watch reads 4 : 30. If the minute hand (a) 10 km (© 5km points East, in which direction will the hour hand point? (a) South-East (b) North-East () North (d) North-West Scanned by CamScanner 6. A man walks 6 km South, turns left and walks 4 km, again turns left and walks 5km. Which direction is he facing now? [SSC (Moltitasking) 20141 (@) South (©) North (©) East (@) West 7. Ram cycled 10 km Southward from his home, tumed right and cycled 6 km, turned right cycled 10km, tumed left and cycled 15 km. How many kilometres will he have cycled to reach straight home? [SSC (CGH 2013), (@) 10 km (b) 21. km (©) 16km (d) 20 km 8. One day, Ravi left home and cycled 10 km Southwards, turned right and cycled 5 km and tumed right and cycled 10 km and turned left and cycled 10 km. How many kilometres will he now have to cycle ina straight line to reach his home? IMAT 2013] (@) 10 km (©) 20km (©) 15 km (@) 25 km. 9. A man travels 4 km due North, then travels 6 km due East and further travels 4 km due North. How far he is from the starting point? [SSC (CG) 2013), (@) 6km (0) 14km (©) 8km (@ 10km 10.A professor leit his college and proceeded towards North for 4 km, after seeing a signboard from point ‘O'. On reaching the place, he saw an another signboard and proceed 4 km to his right to the point ‘'P’, only to find that the signboard was on right of 'P’. From point 'P', he proceeded right to reach that place 4 km away. In which direction he has to go to reach that place from where he has left? MAT 2012) (@) East (©) North (b) West (@) South yoursmahboob.wordpress.com 76 How to Crack Test of Reasoning « Verbal 11, Raju starts from a place P towards North and reaches place Q. From there, he tums towards North-West and reaches place R He, then tums towards South-West and walks toa place S. From there, he turns towards North-West and finally reaches place T. Which of the following figures shows the movement of Raju? T a oT @ Ss © Pp a * Roy oO. 8 a © Ss @ P T P 12. One evening before sunset, two friends Raman and Arjun were talking to each other face to face. If Raman's shadow was exactly to his left side, which direction was Arjun facing? [SSC (Steno) 2013], (0) East (6) South (a) West (©) North 13. A villager went to meet his uncle in another village situated 5 km away in the North-East direction of his own village. From there, he came to meét his father-in-law living in a village situated 4 km in the South of his uncle's village. In what direction is he from initial Position or starting point? (@) North (b) East (c) West (0) South 14. From his house, Dipak went 25 km to North. Then, he turned West and covered 15 km. Then, turned South and covered 10 km. Finally, turning to East, he covered 15 km. In which direction is he from his house? {SSC (CPO) 2013] (@) North () South (c) West (6) East >canneg vy Vamocanner |. Gaurav walks 20 m towards North, the, ss te tam left and walk 40 m. He a tums left and walk 20 m. Further moves 20 m after turning to the right How far is he from his original position? IMAT 2012) (0) 30 m (a) 20m (6) 60m (©) 50m 16, Anamika who is facing South walks 20 m, then she turns to her right ang walks 15 m, then she turns to her right and walks 16 m, again she turns to her right and walks 12 m. How far is she from her starting point? [CG FSC 2013, (@4m (b) 16m (12m (7m (5m 17. Raj starts from his office facing West and walks 100 m straight, then takes a right turn and walks 100 m. Further, he takes a left turn and walks 50 m. In which direction is Raj now from the starting point? [cbs 2013 (a) North-East (b) South-West (c) North (d) North-West 18, Manoj starts from his house and moves towards North. After walking a distance of 4km, he tums left. After walking a particular distance, he reaches Sunita’s house. Sunita’s house is at distance of 5 km from Manoj's house. Then, Manoj walked a total distance of {EPFO 2011] (@) 1 km (b) 5 km (7 km (3 km (€) None of these 19. A gill is facing North. She turns 180° in the anti-clockwise direction and then 225° in the clockwise direction. Which direction is she facing now? {cc psc 2013] (a) West (b) North-East (C) South-West (0) East (©) North-West 20. A man is performing yoga with his head down and legs up. His face is towards the West. In which direction, will his left hand be? IRRB (ASM) 2004) (a) North (©) South (©) East (0) West yoursmahboob.wordpress.com Chapter 7 Direction Sense Test 77 21. Starting from the 12m North, he tumeg nt, Balu walked 24 p, 10 m, he again turned nor 224 walked ‘am and Shyam start walking towards d right Ne 12m, then he turmeq nght and walked forth and cover 20 m. Ram i 5m. How far se left and walked and Shyam to his right. ‘ater 2 iin direction from the starting pe in which Ram walks 10 m in the same direction in (@) 27 m towards East (b) 5 m toe, which he tumed. On the other hand, towards E: ; Geeta Bins Sal 7 etn yam to his right. 22. A child i was looking for his father, He Both walk 25 m forward. How far is Ram went 90 m t rent 90 m towards East before turing to from Shyam now? 188C (CG 2013} Sacisgr to be right aga 20 sy before oo in to look i father at his uncle's place 30 m fom tees am point. His father was not there. He then 5m went 100 m to the North befo: s re his ae ina street, How far fend 25. The houses of A and B face each other on int did the son meet a road going North-South, A's being on father? ree the westem side. A comes out of his house, tums left, travels 5 km, turns (@) 80m ©) 100m (c) 140m (6) 200m right, travels 5 km to the front of D 5 0 the front of D's 23. 1am facing South. 1 tum right and walle house, B does exactly the same and 29m. Then, turn right again and y walk reaches the front of C’s house. In this en, It nN context, which i then turning right walle 20 m.Then, {turn Statement ie connec sar 208 right again and walk 60 m. In which direction am I from the starting point? (@) C and D lve on the same street IRRB (GG) 2012) (0) C's house faces South (@) South-East (b) NorthWest (6) The houses of C and D are less than 20 km apart (©) South-West (d) North-East (6) None of the above Expert Level Exercise 1. After starting from a point, A walks 3km 3. Rajnikanth left his home for office in car. towards East, that turning to his left he He drove 15 km straight towards North moves 3 km. After this he again turns left and then tured Eastwards and covered ‘ ich direction is"A' 8 km. He then tumed to left and covered and moves 3 km. In which direction is 4 am: He again turned left end drove for from his starting point? [ssc 19+2) 20131 20 km and reached office. How far and in (@) North (©) East What direction is his office from the (c) West (@ South home? {18 ACIO) 20131 2. A man starts from his house and walks (@) 21 km West 10 km in South direction, then he turns © tSien Na Sot i in he turns right () forth West right and goes 6 km, again he g (2 20m Noes and goes 10 km and finally tumns right and goes 6 km. At what distance, is he 4. Ram goes 15 m North, then tums right from the starting point and in which Ra ealks 20m, then again turns right direction? ‘and walks 10 m, then again turns right (a) 2km, North (b) 3 km, South and walks 20, m. How far is he from his (©) Atthe starting point (4) 4 km, East original position? {SSC (1062) 2013] (e) None of these as (©) 10 (015 (6) 20 Scanned by CamScanner yoursmahboob.wordpress.com ™ 78 How to Crock Test of Reasoning «Verbal 5.Ram and Shyam start walking in Dil opposite directions, Ram covers 6 km and Shyam 8 km. Then, Ram turns right and walks 8 km and Shyam turns left and walks 6 km. How far everyone is from their starting point? [SSC (CG) 2012), (a) 11 km (b) 8 km (o) 9km (8) 10m 6. Sherley starting from a fixed point, goes 15m towards North and then after tuning to his right, he goes 15 m. Then, he goes 10, 15 and 15 after turning to his left each time. How far is he from his starting point? I8s¢ (CG 2011) (a) 19m (5m () 10m (6) 20m 7, Alok walked 30 m towards East and took @ right turn and walked 40 m. He again took a right turn and walked 50 m. Towards which direction is he from his starting point? (mar 2013] (@) South (0) West (c) South-West (0) South-East 8.One morning at 7 O'clock, Naresh started walking with his back towards the Sun. Then, he tumed towards left, walked straight and then turned towards right and walked straight. Then, he again tured towards left. Now in which direction is he facing? {a} North (b) East (c) West (0) South 9. Anoop starts walking _ towards South. After walking 15 m, he tums towards North. After walking 20 m, he turns towards East and walks 10 m. He, then turns towards South and walks 5 m. How far is he from his original position and in which direction? (a) 10m, North (0) 10m. South (c) 10m West (o) 10m East Scanned by CamScanner rections (Q. Nos. 10-11) Stugy, information and answer the given cueston US ant Point A is 11 m North of point B®, Point C is 11 m East of point B, Point D is 6 m North of point C, Point E is 7 m West of point D, Point F is 8 m North of point E, Point G is 4 m West of point F. 10. How far is point F from point 4? (43m (by 4m 3m @7m (5m 11, How far and in which direction is po, from point A? Point (@) 3m, North (0) 5m, North (¢) 4m, North (0) 4m, South (e) 3m, South 12, From his house, Lokesh went 15 km the North. Then, he tumed West ang covered 10 km. Then, he turned Sout and covered § km. Finally, turning i, East, he covered 10km. In which direction is he from his house? ns, west (new? (a) East {(@) None of these (d) South 13. A man walks 1 km towards East and then he turns to South and walks 5 km. Again he tums to East and walks 2 km, after this he turns to North and walks 9 km How far is he from his starting point? LUBPS (Clee) 2011 (@) 3 km (b) 4 km (©) 5 km (7 km (€) None of these 14. A postman was returning to the post office which was in front of him to the North. When the post office was 100m away from him, he tured to the left and moved 50 m to deliver the last letter at Shanti Villa. He, then moved in the same direction for 40 m, turned to his right and moved 100 m. How many metres was he away from the post office? uc apo) 201) (@) 50 (b) 90 () 150 (@) 100 (©) 120 yoursmahboob.wordpress.com 15. Village Chimur is 20 km to the No: village Rewa. village Rahate is 18 im to the East of village Rewa. village Angne is 12 km to the West of village Chimur. If Sanjay starts from village Rahate and goes to village Angne, in which direction he has to go from his starting point? (a) North (b) North-West (6) South (@) South-East 16. Rohit walked 25 m towards he tured to his left and walked 29 oe an then turned to his left and walked 25 x He again tumed to his right and walked 15 m. At what distance is he from the starting point and in which direction? [UP Bed. 2012 (a) 35 m, East (©) 35 m, North , (0) 40 m, East (A) 60 m, East 17. Ms A goes for her morning walk at 6 O'clock towards the sun for 2 km, then she turns to her right and walks 3 km. She again turns to her left and walks 2 km, finally she tums to her left to walk another 6 km. In which direction is she moving and at what distance from the last turn, is she standing? (@) 6 km, East (b) 9 km, East (©) 6 km, North (@) 9km, North 18. Starting from a point 'S', Mahesh walked 25 m towards South. He turned to his left and walked 50 m. He then again tumed to his left and walked 25 m. He again tumed to his left and walked 60 m and reached a point * from point ‘S' and in which direction? (a) 10 m, West (b) 25 m, North (c) 10 m, East (d) 25 m, West Scanned by CamScanner How far Mahesh is « Chapter 7+ Direction Sense Test 79 Directions (. Nos. 19-20) Study the information and answer the questions given below. On a playing ground, Dev, Kumar, Nilesh, Ankur and Pintu are standing as directed below facing the North. () Kumar is 40 m to the right of Ankur. Dev is 60 m to the South of Kumar. (iii) Nilesh is 25 m to the West of Ankur. (iv) Pintu is 90 m to the North of Dev. 19. If a boy walks from Nilesh, meets Ankur followed by Kumar, Dev and then Pintu, how many metres has he walked, if he has travelled the straight distance all through? (@) 215m (©) 155m (©) 245m (0) 185 m 20. Who is at the North-East of the person, who is to the left of Kumar? (@) Nilesh, (b) Ankur (©) Pintu (6) Either Nilesh or Dev 21. The door of Aditya’s house faces the East. From the back side of his house, he walks straight 50 m, then turns to the right and walks 50 m again. Finally, he turns towards left and stop after walking 25 m. Now, Aditya is in which direction from the starting point? —_[sB1 (PO) 2011] (2) South-East (0) North-East (©) South-West (@) North-West (©) None of these 22. Lakshman went 15 km to the West from his house, then turned left and walked 20 km. He then tured East and walked 25 km and finally turning left he covered 20 km. How far is he now from his house? (@) 15 km (0) 20 km (©) 25km (10 km yoursmahboob.wordpress.com Answer with Explanations A Base Level Exercise 1. @) The movement of man is shown in the figure below N NW NE w € sw ‘SE s From figure, finally, he is facing in the OS direction, which is South-West. 20 N \ E The, pointer which was showing last started ‘showing South Atter accident E> Won Sow NE Pointer turned 90° clockwise. 3. (d) The movement of policeman is shown in the figure below. B 8 For reaching Policeman will Girection, 4k A (starting point) from c, have to move in the West 4 Q N Dah fs Ma Pp s Let and B started from point 0. A tow vards west ang 2 ‘owards East and at last they reached N a? + Scanned by CamScanner =5km 0a=ON | \ oa! 5. (6) 1ce, when the minute hand points tothe hour hand points to the North-East Sirecigg int A and, 6. (6) Let the man starts from the point through Passing! 5 6Km Km Ilo s eakm Band C, he reaches D. Clearly, he is now North. 7. (6) Figure is drawn as — 1skm Dp A C 6km B Now, to go back home Ram has to cycle = (D+ DA)= (15+.6)=21km 8 Now, distance Ravi have t oe is D+ DA) 4a = (10+ 5)=15km yoursmahboob.wordpress.com pone AD yhageas asa nae Now, AD? = AE? + DE? AD? = 6 + (8 AD = (36+ 64 = V100 = 10 km 10. (b) The movement figure of professor is as shown For reaching college from point Q professor will have to move in the West direction. L R N a 3 w £ 8 This situation is traced by option (a). 12, (9) According to the question, Fema —* Raman's Shadow (West) t Agjun 11. @) 8 Hence, Arjun was facing North. 13. (6) Starting point Hence. villager is in the East direction of his initial position. ‘Scanned by CamScanner Chapter 7+Direction Sense Test 81 186m to1n it oem ees A wf i noise) 5 ‘Starting from the point A and passing through the points B, C and D, Deepak reaches the point , Hence, he is in North direction trom his house. 14.) 15, (d) The movements of Gaurav is shown in figure below c 40m 20m E 20m 0 Clearly, Gaurav’s distance from his initial position A= AE = (AD + DE) AD = BC = 40m AE = 40420 A 16. (@) Tem AE = (AF + (EF) = ar + OF =V16+9 v2 AE=5m She is 5 m away from starting 18. (¢) itis clear trom the figure that itis a right angle tnangle. Sunta's "house Sim kn Manoj's house So, by Pythagoras theorem, Fees = =3km ‘Thus, Manoj travelled a distance of 7 Km. 19. (0) 20. (a) N Left hand Head ——>E Right hand s Hence, it is clear from above figure that the left hand points towards North. 21. 10m N 12m w E 8 12m a 0m Sm Finishing point B is at a distance of 15 m from starting point A and in the East direction. 030mC AE 1s the distance from the starting point and the eg. mount ‘carined by CamScanner ee koursmebkgobwordpress.com — F = (3600 + 6400 = V10006 = 100 m 23, (d) The movement of the person from A to F, shown in figure va Clearly, the final position is F Which is to hy North-East of the starting point A 24, (c) Data is arranged as Ram _ Shyam 25m: Ram ‘Shyam .. Distance between Ram and Shyam =(10+7)=17m 25. (0 5km D B Skm skm A Cc Skm So, the houses of C and D are less than 20% apart. B. Expert Level Exercise 1. (@)Final point sem North 3 km West East 3km Starting Point South Thus, A is in North din Tae forth direction from his starting © Starting 9_84ma,- Stan y 10km t0km We f Sim 8 From above fi and final poin gure, itis clear thatthe startin ts of the man are same. na 3. —— kn _, one & fo pein | 15 km ‘5 yor” In AAFE, AE = JAF? + EF? = (AB + BFF + ED -FOy = V6" + (12) = 256+ 144 = V00 =20km So, his office is 20 km North-West <— 20m —> I a 16m 4. (0) [Penarone Starting point ‘Thus, Ram is at a distance of 5 m fron’ his starting point Scanned by CamScanner Chapter 7 - Direction Sense Test 83 5.) oom From above figure. we cer 320 that both are standing equal distance trom stern pm Requited distance = (s} + @ (by Pythagoras theorem) = (HE = 06 = 10% Hence, everyone is 10 km far from their starting Point 6. (c) From figure, A is the starting point and F is the ending point of Sheriey, em 5 as}emy jrom on ‘Ih 15m hsm A AE=AB+ BE = 15+ 10=25 m (+ BE = DC = 10) Now, AF= AE~ EF =25~15 =10m Hence, Sherley is 10 m tar from his starting point. 7. (@) The movement of Alok is shown below Staring point Ending BOE 60m Fron figure, itis clear that he is in South-West direc tion from his starting point 8. (6) Naresh walk’s as Naresh 9 (a) 19m where, P — Starting point Clearly, from figure point Q is at a distance of 10m from point P and in the East direction Solutions (2. Nos. 10-11) afte lam [Ee lzmp Ww € E omg 1m ime 10. (e) AG= BG- AB Gime 3ny A (CD+ EF) (aB) = (6+ 8)-11=3m AF = AS = /9+16=5m 11. ( () Point G is in the ‘North’ direction trom point A ‘and AG = 3m (rom above solution) Scanned by CamScanner ursmanvood.wordpre. 84 How to Crack Test of Reasoning «Verbal 12. (@) The movements of Lekesh are as | figure Mon ¢__10Kn a 5k ——.. 18km DO toKm | Cleary, his final postion E is to the N house at A. sma 13. (0) The movernents ofthe man are 28 shoyn me Clearly, OF = BC EF = (OE ~ DF) = (9-5)= 44m, BF=CD=2km AF =AB+BF=AB+CD= (1+ 2).9 +: Man's distance from starting point A, AE = JAF? 4 EF? = (FFF B= 54m 14.) : oom Peto 100m) 100m D 40m 50mB Shanti Vila Ivis clear trom above figure that postran 80 m away from post office. yoursmahboob.wordpress.com 5. (6) 8 15. (0) 12 Chenu ‘Angne 20km 1ekm A Rewa Rahate From figure, itis clear that A and B denote starting and finishing points, respectively" So, Bis the North-West of point A. 16. (@) The movements of Rohit are shown in figure B 20m c Fohit's distance from starting point, AE =AD + DE =BC + DE =204+15 =35m East B 17. 6km 3km 2 km Final position of A is at a distance of 6 km towards North from her last turn 18. (@)t tom S$ 50m 25m 50m Mahesh is at a distance of 10 m away and in West direction from his starting point S. canned by Cam’ Chapter 7+ Direction Sense Test 85 19. (0) Following the instructions as given in the oe = 30m ‘Niesh «25.0 [arr] «40 (Kigma ‘ 60 my we Dey 8 Total distance covered by the boy =25+ 40+ 60+ 90=215m. 20. (©) From the above figure, itis very clear that the person to the left of Kumar is Ankur and Pintu is to the North-East of Ankur. Therefore, Pintu is the right answer. 21. (@) Since, Aditya's house face towards East and he walks from backside of his house, it means that, he starts walking towards West which is shown, as below NW 425 me sm Clearly, Aditya's final position is D which is to the North-West of the starting point A. 22. (d) The movement of Lakshman is shown below North Points A and E show the starting and end Positions respectively of Lakshman. It is clear that E is 10 km away from A. yoursmahboob.wordpress.com wy 8 — Number, Rankingand Time Sequence Test Sequence and Ranking is based on arrangement of characters/personsobie, in a particular order based on some specific characteristics. In this chapter, we deal with questions which are followed wit ; aaauence of numbers, ranks and time, We have to find answers on the given conditions. The importance of such type of questions canno as their presence in a test of reasonii tbe detingy ing is almost certain. Types of Questions In this section, we will deal with three types of ranking and time sequence test whi questions based on num ich are asked i, Type 1 Number Test 7 in various competitive exam, In this type of questions a number a s given and the candidat et of numbers ite are asked to fin conditions. Or a series of digits id out digits following Certain given Illustration 1, How many 5's a re there in the followi Preceded by 7 and followed ing number i oe ig Sequence which are 755945764598756 764325678 ft (b) 2 () 3 (d) 4 Solution (a)Here, 755945764598 Gleveeaz 5 678 Preceded by 7 and followed by 6 So, there is only one such 5 Scanned by CamScanner yoursmanvboo Chapter 8 «Number, Ranking and Time Sequence Test 87 stration 2. How mai . Mi perenthefooning sores cinee eee of which is preceded by an odd number tan not followed by an even numbery 5348971653 208735 aut O12 ws Solution) 8248971653295 543 There ae thee Such even numbers 6,2 are 8 each ‘of which is preceded by an odd numba followed by an even number, umber andl not Directions ‘llustrations 3-4) Foriowing quest asd on the the tre-dat ranean sie 519 364 287 158 g35 Mlustration 3. If the positions of the first and the third digits within each number are a interchanged, which of the following will be the third digit of the second lowest number? @9 5 wr we Mlustration 4. Which of the following is the difference between the second digit of the lowest and the highest of these numbers? @3 ®1 @2 Wo i Solutions (ilustrations 3-4) 3. (0) According to the question, ater the position ofthe first and third digits interchanged, new numbers are 519 = 915, 364 => 463, 287 = 782, 158 > 851, 835 = 538, so second lowest number = 638 and it is actually 835. Hence, third digit of the second lowest number = 5 Type 2 Ranking Test 4.(6) According to the question, Highest number = 835, Lowest number = 158 Difference between the second digit of the lowest and the highest number =~ 3=2 Mustration 5. The positions of how many digits in the number 351462987 will remain unchanged after the digits are rearranged in ascending order within the number? (@) None (0) One (© Two (@) Three Solution (c) Givennumber 3.5 Ae 29) ‘Aer tearrangement, 1 2al4]5 6 7| Hence, there are two numbers, 4 and 8 remain Unchanged alter the rearrangement. Mlustration 6. The positions of thefirst and the sixth digit in the number 5109238674 are interchanged. Similarly, the positions of the second and the seventh digit are interchanged and so on. Which of the following will be the third digit from the right end after the rearrangement? @9 (0 6 @3 Solution (b) Given number = 5109238674 ‘According to the condition given in question, After interchanging the digits, new number = 9867451092 Hence, third digit from the right end = 0 In this type of questions, generally, the ranks of a person both from the top or from the bottom or the total number of persons are to be found out based on condition given in question, Llustration 7. In a class, boys stand in a single line. One of the boys is nineteenth in order from both the ends, How many boys are there in the class? (a) 27 (b) 37 (38 (d) 39 Solution (b) 19th from left 19th from right Clearly, number of boys in the row = (18+ 18+ 1)=37 The question can be solved by formula also. Total number of persons in a row or class = (Rank of a person from upper end or left end) + (Rank of that person from lower or right end) ~ 1 (because that particular place counts two times) Total number of boys in the clas: 9+ 19-1 37 ee ee Scanned by CamScanner yoursmanbood. wor How t0 Crack Test of Reasoning « Verbal 88 Miustration Anu and Vinay are ranked Any’s rank trom the bottom = 31 — seventh and eleventh respectively from the Tey iy ark tom ho ottom = 3441" 1 {0p in 4 class of 31 students. What will be their rePective ranks from the bottom in the Ilustration 9. In a class of 42 " clay? Mahesh’s rank is 16th from the botton: (a) 20th and 24th (b) 24th and 20th is his rank from the top? i <) 25thand 21st (d) None of these (a) 25th | Number of students ater Anu mn the class () 26th 1 © (B1-7)= 26 (c) 24th NOLS rank hom bottom = 24+ 1.2 25 (@) 27th a 25th from the bottom. Solution (¢) Number of students ahead of Mi De" of students after Vinay nthe class = 42 ~ eee (31-1)=20 Now, Mahesh’s rank from top = 26 + 1 = 97 Now. Vinay’ tank thom bottom = 20+ 1=21 So, Mahesh’s rank is 27th from the top, ny 15 21st from the bottom This question can be solved by formula agg [Note 1 wil be added, « question is ask about another side Rank of a person from upper ot lett {and one sie 1s alroady given number of persons in row) (Rank of tha Question can be solved by formula also. from lower or right end) + 1 lower 7 Alternate Faank of a person ftom lower or fi = (Total numer ot persons tow) (Rank tet Doe Mahesh’s rank from the top = 42 ~ 16, ‘hom upper or left end) + 1 =27 Type 3 Time Sequence Test This t st Is designed to test a candidate's Mustration 11. If every second ability in understanding ‘time sequences. To solve such type of problems a candidate must have a Proper knowledge regarding calender clock Saturdays: all Sundays are holidays in a 30 days beginning on Saturday, then how working days are there in that month? (a) 20 (21 (© 22 (@) 23 Mlustration 10. Sunita leaves her house at Solution (0) As the month be; 20min to seven in the morning, reaches gins on Saturday. So, Sth, 16th, 231d, 30th days are Sundays, whie Vineeta’s house in 25 min, they finish their and 22nd days are second Sat breakfast in another 15 min and leave for turdays. Thus, ate 7 holidays in all their office which takes another 35 min. At ~ Number of working days = 30-7 =23 i cs what time, did they leave Vineeta’s house to Important Facts reach their office? |. A endian yar ‘and (a) 7:40am (b) 7:20am * Ak ns “ (7:45am (4) 8:15 am erenienb le Solution () Sunita leaves her house at 6 - 40 am | * Alleap year is divisible by 4. She ches Vineeta’s house in 25 min, ie * century to bea leap yeay it must be divisible by 400 6. 40+ 0.25 at 7 : 05 am. Both leave for office, * February in a leap year has 29 days. \Smin after 7:05 am ie, 7:05+0:15 4 | « 7: 202m The last day of a year is always the same as the fis da - ' Scanned by CamScanner yoursmahboob.wordpress.com Let us Practice A. Base Level Exercise 1, How many ‘8° are followed by even Henber and preceded 7. unita is the 11th from either end of a row by an odd ft gurls. Hi ny girls are there in that 184381483287849564784186 , secon? @s 4 09 ws (19 2A) 2. What should come next in the follo 8. In a row of boys, Rajan is 10th from the number series? ie al right and Suraj is 10th from the left. 453215453214453213453 ‘When Rajan and Suraj interchange their 2A positions, Suraj will be 27th from the left. (ay (0) 4 (2 (5 Which of the following will be Rajan’s 3. What should come next in the folk position from the right? series of numbers? ——— (a) 10m) 26h (© 2m 223234234523456234567234 (@) 25tn {e) None of these 567 9. In a class of 45 students, a boy is ranked (a2 (b) 3 (4 20th. When two boys joined, his rank was (7 (@) None of these dropped by one. What is his new rank (CSAT 2013), 4, How many 9's are followed by and from the end oem « preceded by numbers divisible by 2? (@) 25th (b) 26H {tPF 20111 (arm (6) 2a 89653596834965269737294137941734 10. Sohan ranks seventh from the top and 98453976 153195742968532957489451 twenty-sixth from the bottom in a class. (a) 12 (06 os How many students are there in the @ 10 @4 class? [SC (1042) 2013) KI (o) 32 33 coy 5. If two is subtracted from each odd digit @ bid @ 6 and three is added to each even digit in 11. There are 25 boys in a horizontal row. the number 3675249. How many digits Rahul was shifted by three places will appear twice in the new number thus towards his right side and he occupies formed? [SSC (Steno) 2012} the middle position in the row. What was his original positon from the left end of @ we @s ae the row ? ICGtPSO 2013) 6. How many such 5s are there in the {a) 15th (b) 16th {c) 12th following number sequence, each of (d) 10th {e) None of these which is preceded by 3 or 4 but not inlored Bort 12 ep an eat een a Temain unc! \g' 959545535845679575545235 after the digits are rearranged in ascending order within the number? (a) Six (b) Three (©) Four BPS (PO) 2017! (6) Five (@) None of these (a) None (b) One (2) Two (6) Three (e) More than three Scanned by CamScanner yours ing - ver" 80 How to Crack Test of Reasoning ow many digits in the remain unc! ge number oF ding order (from tiers (cle) 20721 (0) One fe) 190 (e) More than three jons of hi 19. Nimber 837912 1 after the digits wit rearranged in descen left to nght)? @) None o Twee nan 14, There are twenty people eT orks office. The first group, of Ave wore between 8: 00 am to 2 : 00 Pm. Sees and the third 00 am to 4: group of five works between 12 noon to6 pm. There are three computers in th office which all the employees frequently Use. During which of the following hours the computers are likely to be used most? 1RRB (ASM) 2012] (a) 10.00 am-12 : 00 noon (&) 12 n00n-2 00 pm (©) 1 00pm 00 pm (@)2 00 pm 00 pm In a row of girls, Veena is 12th from the start and 19th from the end. In another row of girls, Sunita is 14th from the start and 20th from the end. How many girls are there in both the rows together? (72 6 «= epg Gt 16. Consider five people A, B, C, D and EF, each having different age. A is younger than only B. C is older than D. D is not 15. the youngest. Who amongst the following are older than C? {cs 2012} (a) AancB (b) E, Band A (C)Aande (d) EandB 17. How many digits position will rem same in the number 5231698, each of which is as far away from the beginning of the number as when the digits are rearranged in ascending order? (a) None (0) One (9) Two (0) Three (©) More than three Scanned by CamScanner mahbogbh,wordpress.com digits position will rem, mete number 6315784, each” ‘as as many digits between th’ r as when the digits 2 rearranged in descending order? (@) None (0) One (o) Two (d) Three (e) More than three any digits position will rema Same ‘in the number 9431658, each which has as many digits between they, in the number as when the digits a, rearranged in ascending order? (b) One (c) Two (e) More than three 18. Ho" 2A samt which hi in the numbe! 19. (@) None (@) Three irecti the sets of num Directions (Q. Nos. 20-24) Study ben given below and answer the questions. 972 682 189 298 751 {LIC (ADO) 2013 20. Ifone is added to the lowest number ang two is added to the highest number, what will be the difference between the second digit of the smallest number ang. third digit of the highest number? | (a5 )7 9 @s (e) None of these 21, If in each number, first and the last digit are interchanged, which one of the following will be the third highest | number? (a) 972 (b) 682 (©) 189 | (9) 298 (e) 751 | 22, Ifin each number, all the three digits are arranged in descending order, which of | the following will be the third highest | number? (@ 972 (0) 682 ©) 189 (@) 298 (©) 751 o | 23. Ifin each number, second and third digit | ors interchanged, what will be the sum of rst digit of the smallest number and last digit of highest number? (a7 (b) 6 () 9 (8 (€) None of these d yoursmahboob.wordpress.com i ‘Sent 24. one is added to the smaller odd th number and one is subtract, rs pumber and one subtaced tm the following will be obtained, if % digit of that odd “higher nemiene subtracted from the second digit of that ty odd lower number thus formed? eat ae 5 4 tat 3 (2 "Sg, Expert Level Exercise 1, Raja walks slower than Raght Raghu walks as fast as Guru and lesen sn walks faster than Guru. Who walks the fastest? IRBI (Grade ‘B”) 2011) , (@) Raghu (©) Raja 2 (@) Krishna (@) Both Raghu and Guru (@) None of these x 2. If the 5th date of a month is Tuesday, ny what date will be 3 days after the 3rd Friday in the month? [ssc 10+2) 2013) (@ 19 (0) 18 (17 (o) 22 3. If 1st October is a Sunday, then 1st November will be (a) Monday (0) Wednesday 4. Ina queue, Mr X is fourteenth from the front and Mr Y is seventeenth from the end, while Mr Z is exactly in between Mr X and MrY. If Mr X is ahead of Mr ¥ and there are 48 persons in the queue, how many persons are there between Mr X and Mr Z? ICSAT 2013] @e6 7 (8 @s 5. If in a clock, 12 is replaced by 1, 11 by 2, 10 by 3 and so on, then what will be the time in that clock corresponding to ‘twenty minutes past three in the usual clock’? [SBI (Clerk) 2012] (@) Twenty minutes past eleven (0) Forty minutes past ten (©) Forty minutes past eleven (d) Twenty minutes past ten (@) None of the above (0) Tuesday (d) Thursday Scanned by CamScanner Chapter 8«Number, Ranking and Time Sequence Test 91 25. If each of the digits in the number 92561473 are arranged in ascending order, what will be the difference between the digits, which are fourth from the right and third from the left in the new arrangement? (@) One © Two (© Tree (0) Four {@) None of these 6. Samant remembers that his brother's birthday is after 15th but before 18th of February, while his sister remembers that her brothers’ birthday is after 16th but before 19th of February.On which date, of February, is Samant's brother's birthday? @ tem © 19% 7. Praveen correctly remembers that his father's birthday is after 21st May but before 27th May whereas his sister correctly remembers that their father's birthday is after 24th May but before 30th May. On which day in May was definitely their father's birthday? MAT 2012] (©) 18th () 17th (a) 25th (©) 26th (©) 24th (d) 25th or 26th 8. Sangeeta remembers that her father's birthday was certainly after 8th but before 13th of December. Her sister Natasha remembers that their father's birthday was definitely after 9th but before 14th of December. On which date of December was their father's birthday? (@) 0th (b) 11th (0) 12th (@) Data inadequate 9. Sneha correctly remembers that her father's birthday is before 16th June but after 11th June whereas, her younger brother correctly remembers that their father's birthday is after 13th June but before 18th June and her elder brother anbdoo. . wordpress.com q 92 How to Crack Test of Reasoning « Verbal birthday is on an éven date. On what te, is definitely their father's pray [Vijaya Bank (Clerk) 20101 birthday? (a) Sateenth (©) Tweltth (c) Fourteenth (@) Data inadequate {6} None of these 10. Five birds Crow, Pigeon, Little pigeon, Big crow and Eagle fly one after other from a tree branch. Big crow flew after Crow but is ahead of Eagle. Pigeon is between Crow and Big crow. Little pigeon is before Crow. Which bird is in [SSC (Constable) 2012] the last? (a) Pigeon (b) Big crow (c) Eagle (d) None of these 11. If it is possible to make only one such number with the first, the fourth and the sixth digits of the number 531697, which is the perfect square of a two-digit even number, which of the following will be the second digit of the two digit even number. If no such number can be made, give '@' as the answer and if more than one such numbers can be made, give ‘©! as the answer. (a) 4 (b) 2 6 @@ ee 12. In a group of five districts, Akbarpur is smaller than Fatehpur, Dhanbad is bigger than Palamu and Bara Banki is bigger than Fatehpur but not as big as Palamu. Which district is the biggest? (SSC (1042) 2011 (@) Akbarpur (0) Fatehpur , (6) Dhanbad (0) Palamu 13. In a class, Neha's rat top and 30th from the botten Wm the total number of students in that class? wa [Allahabad Bank (PO) 2012) (0) 51 80 (b) 49 (e) 82 Scanned by CamScanner . The positions of the first and fitth g; - the umber 53146872 are interch ty Similarly, the positions of the seconti% the sixth digit are interchanged 0° on, Which of the following will bee w digit from the right end agter * rearrangement? thy @2 (b) 4 6 1 (e) None of these Directions (@. Nos. 15-18) Study the Sets of rum given below and answer the questions, 489 541 654 953 783 15. If in each number, the first and the digit are interchanged, which of “* following will be the second higned number? (a) 489 (b) 541 (c) 654 (d) 953 (e) 783 16. If in each number, all the three digits ate arranged in ascending order, which the following will be the lowest umber? | (a) 489 (o) 541 (©) 654 | (@) 953 (e) 783 17. If five is subtracted from each of numbers, which of the foll numbers will be the difference betw. the second digit of second hig number and the second digit of highest number? (@o 3 (a4 fe) 2 1 18. If in each number, the first and the second digit are interchanged, whic will be the third highest number? (@) 489 (0) 541 (©) 783 (0) 953 (e) 654

You might also like